Endo Q's

Réussis tes devoirs et examens dès maintenant avec Quizwiz!

You are asked to see a 48-year-old Caucasian male who was admitted to the ICU for hypotension after an MVA, when he suffered a broken thumb. The orthopedic surgeon reported a simple fracture and placed a cast on his hand, but the patient lost consciousness a few minutes later and was noted to have a BP of 75/palp. BP is 100/64 while on IV saline. HR in the ER was 130 but is now 80. Temp is 98.7° F. You note that he is thin and tan with sparse axillary hair. His skin is warm and moist. He denies fatigue or cold intolerance. Na: 134 K: 5.6 Cl: 102 HCO3: 22 BUN: 42 Creat: 0.7 Glu: 61 Head CT normal Which of the following is the most appropriate next step in patient care? A) Dexamethasone + fludrocortisone and order an ACTH stimulation test. B) Thyroxine daily. C) Hydrocortisone and thyroxine daily. Order an ACTH stimulation test and TSH. D) Hydrocortisone + fludrocortisone and order an ACTH stimulation test. E) Hydrocortisone.

A. Dexamethasone + fludrocortisone and order an ACTH stimulation test. Explanation The problem list: Hypotension in a tanned male with sparse hair Hyponatremia and hyperkalemia Prerenal state Anion gap = 10 This case is straightforward primary adrenal insufficiency (Addison's), but the question asks you indirectly about methods of diagnosis. The patient is tan because of an elevated ACTH level in response to inadequate cortisol production by the adrenal. The adrenal cortex is abnormal (because of tumor or vascular damage), and aldosterone is also deficient, causing the hyponatremia and hyperkalemia. (In cases where the defect is in the pituitary and ACTH is not formed [secondary adrenal insufficiency], patients are not tanned, and they do not have the sodium and potassium derangements because the adrenal secretes aldosterone perfectly fine.) Clearly, the option to address only the thyroid gland is wrong. The answer with hydrocortisone instead of dexamethasone and fludrocortisone asks you to 1) differentiate between methods of diagnosis; and 2) include treatment for the comorbid mineralocorticoid deficiency. This question is included specifically to teach you about ACTH-stimulation tests and query whether you understand that primary adrenal disease includes both aldosterone and glucocorticoid deficiencies. Empiric therapy should include dexamethasone if you plan to try to diagnose Addison's with an ACTH-stimulation test, because every steroid except dexamethasone cross-reacts with the assay. The fludrocortisone replaces the missing mineralocorticoids. Patients who are clearly hypothyroid should never receive thyroid replacement if they are also adrenally insufficient (Schmidt syndrome) until after cortisol has been replaced. If you give thyroid replacement first, patients can die quickly due to an inability to respond to the increased metabolic demand. The clinical scenario in this question, however, gives no indication that the patient has comorbid hypothyroidism.

A woman comes to your clinic complaining of weight gain and depression. On physical examination, you find moon facies, cervicodorsal fat pad (buffalo hump), central obesity, plethora, and some bruises. She tells you that she is depressed. You suspect Cushing syndrome and begin the workup. Which of the following is true? A) A normal 24-hour urine cortisol (10-90 µg/day) essentially excludes Cushing syndrome. B) An overnight dexamethasone suppression test essentially excludes Cushing syndrome if the morning cortisol is 6-10 µg/dL. C) A low-dose overnight suppression test uses 0.5 mg of dexamethasone. D) An elevated 24-hour urine cortisol diagnoses Cushing disease. E) A low-dose overnight suppression test uses 2 mg of dexamethasone

Answer A. A normal 24-hour urine cortisol (10-90 µg/day) essentially excludes Cushing syndrome. Explanation This question requires you to know the initial workup of suspected hypercortisolism. This patient has clinical findings that are suspicious for Cushing syndrome. However, these clinical findings are also associated with other disorders—or can be nonspecific. The first step of the workup is to verify hypercortisolism. This can be done in two ways. An elevated 24-hour urine cortisol indicates excessive cortisol and is compatible with Cushing syndrome, but it is not diagnostic of Cushing disease (pituitary tumor secreting excess ACTH). However, a normal 24-hour urine cortisol excludes all but the rarest types of Cushing syndrome. The other test for hypercortisolism is the low-dose overnight dexamethasone suppression test. This test involves the patient taking 1 mg of dexamethasone at 11 p.m. and having serum cortisol measured at 8 a.m. the following morning. A normal person will have their morning cortisol suppressed to < 5 μg/dL.

A 28-year-old woman takes paroxetine for depression. She was diagnosed when her family suggested she see a therapist because she lacked interest in her usual activities. Six months later, the therapist asked for evaluation because there was no improvement on this antidepressant nor with the 2 other antidepressants tried prior. She has lost 12 pounds, which she associates with multiple flu-like episodes she's experienced over the preceding 18 months. BP 94/54, P 98 She is thin and pale. She has no tremor or diaphoresis and otherwise has no focal findings. She has a family history of thyroid disease and has been having GI symptoms during this time period. Which of the following should be the next test you order? A) Basal plasma ACTH and cortisol ACTH stimulation test B) Dental evaluation for etched enamel C) MRI of head D) T3 E) CBC

Answer A. Basal plasma ACTH and cortisol ACTH stimulation test Explanation This is a classic presentation for Addison disease. She has fatigue, asthenia and listlessness, and recurrent flu-like illnesses. Further history may be that of a family history of autoimmune endocrine disorders (Type 1 diabetes, Hashimoto disease, Graves disease) or of tuberculosis she acquired overseas (if it had been left untreated). A basal plasma ACTH and cortisol ACTH stimulation test should be done next. If the cortisol is low and the ACTH is high, it indicates primary adrenal insufficiency (Addison disease). Screening for other autoimmune endocrine disorders is reasonable, but not a priority. This is not the history of bulimia, so a dental screening for enamel etching due to vomiting is not necessary. Treatment should be a corticosteroid and mineralocorticoid, since the aldosterone-producing cells are often involved to some extent.

A 49-year-old man presents for a follow-up exam. He was diagnosed with Type 2 diabetes 10 years ago. Current therapy includes metformin 500 mg bid and glyburide 5 mg bid. He is also treated for hypertension with hydrochlorothiazide 25 mg/day. He does not test blood glucose at home. He feels well. BMI = 44; BP is 145/90; he has no edema. Funduscopic exam is unremarkable. Laboratory: Serum creatinine 0.8 mg/dL (0.5-1.4 mg/dL) Hemoglobin A1c 8.8% (4-6%) Fasting plasma triglycerides 1,077 (< 150 mg/dL) Urine microalbumin-creatinine ratio 131 (0-30 mcg/mg) Which of the following is the best set of adjustments for his therapeutic regimen? A) Begin enalapril, begin fenofibrate, and begin insulin glargine. B) Increase glyburide to 10 mg bid, begin lisinopril, and begin pravastatin. C) Begin 70/30 insulin bid, stop metformin, and increase HCTZ to 50 mg/day. D) Stop glyburide, begin 70/30 insulin bid, begin amlodipine 5 mg/day. E) Increase metformin to 1,000 mg bid, increase glyburide to 10 mg bid, and begin gemfibrozil.

Answer A. Begin enalapril, begin fenofibrate, and begin insulin glargine. Explanation Key points: The patient requires better control of hyperglycemia, initiation of an ACE inhibitor for control of hypertension associated with microalbuminuria, and initiation of therapy for severe hypertriglyceridemia. The latter poses a danger of causing acute pancreatitis. Increasing metformin to 100 mg bid is appropriate but will not make a major impact on HbA1c. The maximal effective dose of glyburide is 10 mg per day. Either gemfibrozil, fenofibrate, or niacin would be appropriate therapy for hypertriglyceridemia. Until the triglycerides are lowered to < 400 mg/dL, LDL cannot be calculated, and thus, the need for a statin is unclear. Although a third oral agent, such as a TZD or a GLP-1 agent, could be added to metformin and glyburide, the addition of nightly basal insulin, such as detemir, glargine, or NPH, is more likely to lower HbA1c to < 7.5%.

You are asked to see an elderly woman with CHF in the CCU. The cardiologist tells you that the patient just isn't responding to treatment like she should. You find her somnolent but easy to arouse and in good spirits. She reports recent fatigue and edema at the nursing home. Your examination reveals normal body hair but dry skin. She has lower extremity and periorbital edema. Heart rate is 64 with a third heart sound. The lungs have bilateral anterolateral crackles. A recent echo showed some left ventricular wall abnormalities and a mildly decreased ejection fraction, but no pericardial effusion. You order thyroid tests, which come back as the following: FT4 0.3 ng/dL (0.7-1.5 ng/dL) TSH 58 mU/L (0.5-5.0 mU/L) What is your recommendation at this time? A) Begin oral thyroxine 12.5 µg per day while closely watching her cardiac status. B) Oral thyroxine 200 µg per day and oral triiodothyronine (T3) 25 µg per day. C) Check reverse T3 (rT3) for suspected euthyroid sick syndrome. D) Begin intravenous thyroxine therapy for rapid return to normal. E) Begin intravenous thyroxine and triiodothyronine (T3) to replace the missing T3 and to saturate the unused binding sites of thyroxine binding globulin.

Answer A. Begin oral thyroxine 12.5 µg per day while closely watching her cardiac status. Explanation This question requires you to understand the potential dangers of treating hypothyroidism in someone with pre-existing heart disease. This patient is clearly hypothyroid and needs to be started on thyroid therapy. Correction of her hypothyroidism should benefit her cardiac condition—but could also precipitate a serious acute cardiac event. Therefore, anyone with pre-existing cardiac disease should be treated cautiously. Her thyroid status should be corrected slowly while watching her closely. She likely has coronary artery disease, so a sudden improvement in her cardiac output will be accompanied by an increase in O2 demand, which impaired coronary arteries may not be able to provide. The best solution is to replace her thyroid hormones slowly, such as 12.5 μg of thyroxine per day. Any use of triiodothyronine or intravenous thyroxine will replace her thyroid hormones too quickly. Also, though she is very ill, this is not a case of euthyroid sick syndrome.

A 60-year-old man comes to your clinic complaining of weight loss and reports losing 25 pounds over the last year. He recently moved to your town hoping that a change of pace might make him feel better. He has been somewhat depressed for the last 2 years. He doesn't get much exercise and often feels tired. He has smoked about a pack a day since he was in his twenties. He admits to feeling stressed in your office because he hasn't seen a doctor for over 20 years. His affect is somewhat flat, and his pulse is 101. Otherwise, your examination is unremarkable except for a mild tremor on outstretched extremities. Because of his smoking history and unexplained weight loss, you suspect lung cancer and order a chest x-ray. He returns for follow-up and is unchanged from the earlier office visit. The radiologist reports the chest x-ray as normal. Which of the following should you do next for this patient? A) Check FT4 and TSH. B) Begin an antidepressant and have him return to clinic for follow-up. C) Refer him to an oncologist for workup of an occult cancer. D) Refer him to a psychiatrist for evaluation of depression.

Answer A. Check FT4 and TSH. Explanation Many things can cause an unexplained weight loss, and depression is common in older patients. However, these symptoms are also associated with hyperthyroidism. Unfortunately, older individuals do not always exhibit the adrenergic response to hyperthyroidism. The term apathetic hyperthyroidism refers to hyperthyroidism without the usual manifestations of palpitations, heat intolerance, etc. The only sign exhibited by the patient in this question is tremor and tachycardia. Unexplained weight loss with a long history of smoking should always make you suspicious of cancer. Most physicians would order a chest x-ray. When it comes back normal, one should rethink the case. The thyroid should be evaluated before more extensive screening for cancer takes place. Depression is common, and it may be appropriate either to begin treating him or to refer him to a psychiatrist. However, do not forget that hyperthyroidism can cause depression. If the patient has hyperthyroidism and depression, it is not appropriate to treat the depression while ignoring the hyperthyroidism. Treat the hyperthyroidism and follow the depression; the depression will often resolve once the patient is euthyroid.

A 28-year-old male is involved in a motor vehicle accident while riding a motorcycle and experiences loss of consciousness, despite wearing a helmet. He regains consciousness while en route to the hospital and denies any sequelae. On arrival at the hospital, the emergency department staff notes a mildly obese young man with central obesity, blood pressure 160/90, and multiple mild lacerations, but otherwise unremarkable. Routine labs are within normal limits. Other than general soreness, he has no complaints. A CT of the head is normal except for a 3-mm well-visualized pituitary mass. He is discharged from the emergency department after a period of observation and comes to see you several days later for follow-up. Your exam is similar to the emergency department's except that now his blood pressure is 140/85. Which of the following would be the most appropriate next step at this time? A) Check serum IGF-1 and 24-hour urine cortisol. B) Discharge him from your clinic on a drug to treat hypertension and encourage him to stop riding motorcycles. C) Check an ACTH stimulation test, TSH, and a total testosterone. D) Refer him to a neurosurgeon for transsphenoidal resection of the pituitary mass. E) Have him return to your clinic in a few weeks to recheck his blood pressure.

Answer A. Check serum IGF-1 and 24-hour urine cortisol. Explanation Whenever a pituitary incidentaloma is discovered, two main questions must be answered. One, is the mass inhibiting secretion of any pituitary tumors? Two, is the tumor secreting something? Let's review the inhibiting scenario first. Many pituitary tumors do not appear to secrete anything and are called null cell tumors. With the advent of sensitive assays, it has been learned that some of these null-cell tumors actually do secrete. They may be secreting α-chains or gonadotrophins (LH or FSH), which rarely cause symptoms. After all, what man is going to complain about larger testes or a higher sperm count? However, some of these tumors get large enough that they inhibit secretion of pituitary hormones because of a mass effect. In this patient, the tumor is a microadenoma (< 10 mm) and is too small to inhibit secretion. The other scenario is one of excess secretion. Does the pituitary tumor secrete any hormones? Remember the main 6 anterior pituitary hormones: ACTH, LH, FSH, GH, TSH, and PRL. A fishing expedition to test all hormones is not recommended. Instead, evaluate the patient for signs or symptoms that may point toward a particular excess such as galactorrhea or loss of libido (PRL); amenorrhea (LH, FSH); weight loss, hyperdefecation, tachyarrhythmia, or heat intolerance (TSH); hypertension, hypokalemia, or alkalosis (ACTH). These findings can be very subtle or even absent in the case of a new tumor or a very small one. If your evaluation does not point toward a particular hormone, then check the adrenal and thyroid axes. A screening test for the adrenal axis is a 24-hour urine for cortisol; a value < 70 μg is normal. The thyroid axis can be checked by a TSH and T4 drawn at any time of the day. You are cautioned to check one more hormone. A small tumor may be secreting GH. The effects of over-secretion of GH take many years to become evident, and many more years usually transpire before it is recognized. These patients develop acromegaly and its many serious complications. Though GH is pulsatile, its effect on IGF-1 is not. Measure a random IGF-1 in any patient with a pituitary incidentaloma. In this patient, the central obesity and hypertension may indicate hypercortisolism (Cushing syndrome). Order a 24-hour urine for cortisol and don't forget to check the IGF-1 level. "Refer him to a neurosurgeon for transsphenoidal resection of the pituitary mass" is incorrect because there are no reasons to refer this patient for surgery at this time. "Return to your clinic in a few weeks to recheck blood pressure" is incorrect because the pituitary tumor must be evaluated and his blood pressure management must be followed. "Discharge him from your clinic on a drug to treat hypertension and encourage him to stop riding motorcycles" is incorrect because, once again, the pituitary tumor must be evaluated. "Check an ACTH stimulation test, TSH, and a total testosterone" is incorrect because hyposecretion of ACTH, secondary hypothyroidism, and secondary hypogonadism are not suspected.

A thin, elderly man slipped in his bathtub and was knocked unconscious. His wife called the ambulance. He recovered before the ambulance got him to the hospital. A CT scan of his head was normal, except for a 2 mm pituitary mass. The physician in the Emergency Department referred him to you for follow-up. Your examination shows nothing to be out of the ordinary except for numerous skin tags and a fasting glucose of 128 mg/dL. He had a screening colonoscopy 8 months ago that found several small polyps that were all benign. A 24-hour Holter monitor failed to show any arrhythmias. Which of the following is the most appropriate next step? A) Check the IGF-1 level because you suspect acromegaly. B) Refer him to a neurosurgeon for possible resection of the mass. C) Recheck the fasting glucose next month for possible diabetes. D) Repeat the head CT in 6 months. E) Check a 24-hour urine cortisol to exclude Cushing disease.

Answer A. Check the IGF-1 level because you suspect acromegaly. Explanation IGF-1 is insulin-like growth factor 1. The emergency physician discovered a pituitary incidentaloma. Though these are somewhat common, you must always evaluate the patient carefully so as not to miss a significant tumor. His tumor may not be benign. He has skin tags, colon polyps, and hyperglycemia, which can all be seen in acromegaly. This is a rare but important diagnosis because of its associated morbidity. The easiest test is the serum IGF-1 level. If it is elevated, he should be referred to a neurosurgeon

A 29-year-old female presents for evaluation of fatigue. She reports decreased energy throughout the day that has progressively worsened over the past 2 months. She notes occasional palpitations and 7-kg weight loss. She denies any recent illness or infection. On physical exam, her HR is 95, and her BP is 145/87. Thyroid exam reveals a smooth, enlarged thyroid gland that is nontender to palpation. Thyroid function testing reveals: TSH - < 0.01 uU/mL (0.5-5.0 uU/mL) Free T 4 - 2.6 ng/dL (0.9-4 ng/dL) Radioactive iodine uptake scan reveals decreased uptake throughout the thyroid gland. Which of the following is the most likely diagnosis? A) Chronic lymphocytic thyroiditis B) Subacute thyroiditis C) Toxic multinodular goiter D) Graves disease E) Exogenous hyperthyroidism

Answer A. Chronic lymphocytic thyroiditis Explanation Chronic lymphocytic thyroiditis (Hashimoto thyroiditis) is the most common cause of hypo thyroidism. However, in its early stages, the autoimmune response results in elevated thyroid hormone levels due to antibody-mediated cell destruction in the gland. The thyroid is typically enlarged and firm, but nontender. Subacute thyroiditis typically occurs after an antecedent trigger like a viral infection and often produces a tender thyroid. Toxic multinodular goiter and Graves disease are additional causes of hyperthyroidism. With both, you would expect to see normal to likely increased uptake on a radioactive iodine uptake scan, indicating increased synthesis of thyroid hormone. Exogenous hyperthyroidism would demonstrate decreased uptake on a scan, but would not produce an enlarged thyroid gland.

A patient of yours comes to the clinic for his routine follow-up. He is 47 years old and has Type 2 diabetes. He is moderately obese, has mild hypertension treated effectively with an ACE inhibitor, and has hypercholesterolemia treated with a statin. Before you started the statin, his LDL-cholesterol was 170 mg/dL. His glycemic control has been difficult, and he is currently taking a thiazolidinedione, metformin, and a rapid-acting secretagogue, all at the maximum dose allowed. Despite 3 drugs, his glycemic control has worsened during the past 6 months along with his weight. He states that he is tired of watching his diet and admits to gaining 21 pounds in the last 6 months. The blood pressure today is 126/74. The physical examination is unchanged; his feet are fine. LABORATORY 6 months ago (fasting): Na+ 140 mEq/L (135-143 mEq/L) K+ 4.0 mEq/L (3.5-5.0 mEq/L) Cl- 107 mEq/L (100-109 mEq/L) HCO3- 24 mEq/L (22-30 mEq/L) Urea nitrogen 11 mg/dL (8-18 mg/dL) Creatinine 1.0 mg/dL (0.6-1.2 mg/dL) Glucose 102 mg/dL (65-110 mg/dL) HbA1c 8.5% (4.9-6.2%) TChol 169 mg/dL LDL 92 mg/dL HDL 38 mg/dL Triglycerides 195 mg/dL Random urine microalbuminuria screen is 12 μg/mg (< 30 μg/mg). LABORATORY today (fasting): Na+ 139 mEq/L (135-143 mEq/L) K+ 4.1 mEq/L (3.5-5.0 mEq/L) Cl- 105 mEq/L (100-109 mEq/L) HCO3- 24 mEq/L (22-30 mEq/L) Urea nitrogen 12 mg/dL (8-18 mg/dL) Creatinine 1.0 mg/dL (0.6-1.2 mg/dL) Glucose 179 mg/dL (65-110 mg/dL) HbA1c 10.2% (4.9-6.2%) TChol 198 mg/dL LDL 98 mg/dL HDL 36 mg/dL Triglycerides 320 mg/dL Random urine microalbuminuria screen is 17 μg/mg (< 30 μg/mg). Which of the following is the best choice to address the hypertriglyceridemia? A) Discontinue his oral agents for glycemic control and switch him to insulin. B) Stop the statin and begin a fibrate. C) Add a fibrate and another statin to his current statin. D) Add an α-glucosidase inhibitor to his diabetic regimen. E) Continue his statin and add a second statin.

Answer A. Discontinue his oral agents for glycemic control and switch him to insulin. Explanation This question requires you to know that triglyceride levels are very dependent upon glycemic control. This patient has gained a significant amount of weight, and his insulin resistance has worsened to the point that he is not controlled on the combination of a thiazolidinedione, metformin, and a rapid-acting secretagogue, despite taking the maximum allowable doses. At this point, he must be switched to insulin. His triglyceride level was much better 6 months ago when his glycemic control was better. It is safe to assume that the increase in triglyceride levels is due to his worsening control. This scenario is commonly seen with diabetic patients. Before specifically targeting his triglyceride levels, his glycemic control must first be improved. Adding a fibrate to a statin is controversial. Though some physicians do it, the combination is not approved by the FDA at the time of this writing. A fibrate could certainly be used once the statin was discontinued, but a fibrate is not as good an agent at lowering LDL cholesterol. His LDL was 170 mg/dL in the past. It is unlikely that his LDL will remain controlled once he has been switched from a statin to a fibrate. An α-glucosidase inhibitor is expected to lower HbA1c by about 0.5 percentage points. This will not provide enough control for his hyperglycemia. Adding a second statin will not help the situation. If anything, this will increase his risk of an adverse event.

A young man comes to your office complaining about breast development. He denies taking any medications or illicit drugs. He has never used marijuana and doesn't even drink. His beard has gotten heavier, and he now has to shave twice a day. The rest of his history is negative, and your physical examination shows only mild bilateral gynecomastia and normal-sized but soft testes. You order some hormone tests and learn that his testosterone, estradiol, and dehydroepiandrosterone (DHEA) levels are elevated and his gonadotropins (LH and FSH) are low. What do you suspect? A) He has an adrenal tumor secreting dehydroepiandrosterone. B) He is taking exogenous testosterone though he denies it. C) He is taking exogenous estrogens though he denies it. D) He has Klinefelter syndrome. E) He has a testicular tumor secreting testosterone.

Answer A. He has an adrenal tumor secreting dehydroepiandrosterone. Explanation This question requires you to recognize an adrenal tumor secreting androgens. He most likely has an adrenal tumor secreting dehydroepiandrosterone (DHEA). The excess DHEA is converted into testosterone, and the excess testosterone leads to higher estradiol levels through peripheral aromatization of testosterone. The next step would be a CT scan of his adrenals and surgery for any adrenal mass discovered. However, this scenario could be caused by exogenous DHEA. He denies it at this time, and it isn't one of the choices. Increased testosterone, either exogenous or from a testicular tumor, would raise the levels of estradiol, but he is not taking exogenous testosterone because it would not increase his DHEA level. He is not taking exogenous estrogen because it would not increase either his DHEA or testosterone levels. Individuals with Klinefelter syndrome do not have elevated DHEA, testosterone, and estradiol levels—and do not shave this often.

A 72-year-old female presents to the emergency department with increasing lethargy following two days of increasing nausea and vomiting. She was diagnosed with a urinary tract infection three days ago and started on a course of oral ciprofloxacin. Her past history is remarkable for a diagnosis of temporal arteritis six months previously for which she has recently completed an extended weaning for oral dexamethasone. On examination, her blood pressure is 112/65 mmHg: temperature is 97.5° F (36.4° C); heart rate 92/min, and respiratory rate 14 breaths/min. She rouses with stimulation and follows simple commands. There is no meningismus and no costovertebral angle tenderness. Urinalysis shows 3-5 RBC and trace protein but is otherwise unremarkable. CBC shows white blood cell count of 9.8 x 10 3 /mm 3 with a normal differential. Of the following, the most important intervention at this time would be: A) IV hydrocortisone B) IV bolus of normal saline C) Drug screen D) CT scan of the head E) Broad-spectrum IV antibiotics

Answer A. IV hydrocortisone Explanation Many patients with rheumatologic disease are treated with courses of steroid medication. This therapy, especially when extended over long periods of time, is a potent suppressant of the adrenal axis. During periods of stress, including infection, surgery or trauma, the adrenal gland is incapable of secreting sufficient quantities of cortisol, and the patient can experience an adrenal crisis. This is manifest with lethargy, nausea and vomiting, and circulatory collapse. If unrecognized and untreated, the patient is at significant risk of death. It is therefore extremely important that any patient who may demonstrate adrenal suppression should routinely be treated with IV corticosteroids during periods of significant stress. This should occur promptly and not be delayed pending results of other studies, such as a drug screen or a CT scan. Although this patient was diagnosed with a UTI, she had already received antibiotics; her WBC count is not elevated; she is afebrile, and the urine is unremarkable for active infection at the time of presentation. Antibiotics may be appropriate, but without steroid supplements, they would not reverse this patient's condition. The same is true for IV fluids. A drug screen may be appropriate in an individual with altered mental status, but by itself would not improve this patient's care. Without focal findings and without history of trauma, a CT scan would also delay the needed steroid supplementation.

A tearful 24-year-old woman comes to discuss her Type 1 diabetes. She is on insulin lispro and NPH. She has increased her NPH from 14 units in the morning and 12 units at bedtime to 16 units in the morning and 16 units at bedtime because she has high morning glucoses. Her log shows FBS 42→325, noon 112→201, supper 68→167, bedtime 189→220. She notes that her sleep is restless, and she has been having increased nightmares and never feels rested in the morning. Which of the following is the next appropriate step in patient care? A) Increase the supper insulin lispro, and decrease the bedtime NPH. B) Make no changes at this time. C) Increase both supper insulin lispro and bedtime NPH. D) Increase a.m. insulin lispro and NPH, and increase bedtime NPH. E) Decrease noon insulin lispro and a.m. NPH, and increase bedtime NPH.

Answer A. Increase the supper insulin lispro, and decrease the bedtime NPH. Explanation You are asked to recognize that erratic fasting glucoses may represent unrecognized nocturnal hypoglycemia, and so you should cut her bedtime NPH.

A 40-year-old female has lost 10 pounds over the last 2 months, complains of excessive thirst, and has repeated vaginal yeast infections. Her BMI is 23, and her exam is normal except for dry buccal mucosa and vaginal discharge. Her fasting blood sugar this morning is 380. Which of the following would you recommend as initial therapy for the diabetes? A) Insulin B) Glipizide C) Metformin D) Pioglitazone

Answer A. Insulin Explanation Symptomatic patients require insulin as initial therapy. Any one of these could be added later, but none of the agents alone will alleviate the symptoms or rapidly lower the blood sugar.

A 27-year-old male presents with renal colic and is found to have nephrolithiasis. The stone is noted to be composed of calcium phosphate. His only medications are a thiazide diuretic and megadose vitamin supplements. After the acute episodes have resolved, an evaluation is instituted and discovers an elevated serum ionized calcium of 3.6 mmol/L (1.1-1.4 mmol/L). A parathyroid hormone (PTH) level of 120 pg/ml (10-65 pg/mL) was obtained. Which of the following is the most consistent with this clinical presentation? A) MEN Type I B) Vitamin D intoxication C) Hyperthyroidism D) Thiazide diuretic

Answer A. MEN Type I Explanation Hypercalcemia is commonly associated with hypercalciuria and a subsequent increased risk for calcium-based nephrolithiasis. An elevated serum calcium coupled with an elevated PTH and hypercalciuria is consistent with hyperparathyroidism. This condition can exist independently and is also one of the components of MEN Type I (tumors of the P ituitary, P ancreas, and P arathyroid glands). Vitamin D intoxication leads to a suppressed rather than elevated PTH level. Hyperthyroidism can cause an increased metabolism of bone and subsequent increase in calcium, but this also causes a decrease in PTH levels. Thiazide diuretics can increase calcium, but usually reduces urinary calcium excretion and also tend to lower PTH levels.

A 21-year-old man comes in for evaluation of polyuria and polydipsia. Here are his initial laboratory values: Serum sodium 144 mg/dL Serum potassium 4 mg/dL Serum chloride 107 mg/dL Serum bicarbonate 25 mg/dL BUN 18 mg/dL Glucose 102 mg/dL Urine sodium 28 mg/dL Urine potassium 32 mg/dL Urine osmolality 195 mosmol/kg water Okay ... You fluid-deprive him for 12 hours and his body weight falls 5%!! Now, now you send for laboratory again: Serum sodium 150 mg/dL Serum potassium 4 mg/dL Serum chloride 110 mg/dL Serum bicarbonate 25 mg/dL BUN 19 mg/dL Glucose 100 mg/dL Urine sodium 24 mg/dL Urine potassium 35 mg/dL Urine osmolality 200 mosmol/kg water Now, you give him 5 units of arginine vasopressin subcutaneously, and his urine values 1 hour later show: Urine sodium 30 mg/dL Urine potassium 30 mg/dL Urine osmolality 199 mosmol/kg water Given these lab results, which of the following is his most likely diagnosis? A) Nephrogenic diabetes insipidus B) Lupus nephritis D) Osmotic diuresis C) Psychogenic polydipsia D) Salt-losing nephropathy

Answer A. Nephrogenic diabetes insipidus Explanation He cannot concentrate urine even with severe hypertonic dehydration—this suggests diabetes insipidus. The nephrogenic origin is suggested when there is no increase in urine concentration after giving exogenous vasopressin.

A 24-year-old female with Graves disease was treated with radioactive iodine 9 months ago. Her weight has returned to normal; she no longer has palpitations or heat intolerance, and her menstrual periods are regular. Thyroid function studies last week revealed a free T 4 of 1.4 (0.9-1.5) and a TSH of < 0.05 (0.2-4.2). Which of the following should you recommend now? A) No intervention B) Another dose of radioactive iodine C) Measure TSH receptor antibody D) Methimazole

Answer A. No intervention Explanation This patient is clinically euthyroid and has a normal FT 4 . In Graves disease, TSH levels may remain suppressed for months after treatment with an antithyroid agent or radioactive iodine, and therapeutic decisions should not be made on the basis of TSH alone. Methimazole and giving another dose of radioactive iodine are wrong because she is not hyperthyroid. While she may have a positive TSH receptor antibody, the antibody titer is not helpful in determining whether she is euthyroid.

A 46-year-old female with a history of diabetes presents obtunded. She has no other medical problems and lives with her 17-year-old daughter. She takes NPH insulin bid. Her daughter states that recently, the patient has been having severe nightmares but otherwise has been in her usual state of health. She underwent menopause from age 38-42. PE: T: 96° F BP: 110/60 HR: 66 RR: 14 Alopecia on the head, pubic region, and eyebrows Arousable to deep sternal rub, reactive pupils Delayed reflexes Na: 122 K: 3.4 Cl: 82 HCO3: 23 Glu: 110 BUN: 40 Creat: 1.0 Hgb: 10.6 MCV: 86 TSH: 0.45 (0.5-5.0) Free T4: 0.75 (0.9-2.4) FSH: 15 (> 35 postmenopausal) LH: 22 (> 30 postmenopausal) Which of the following is the most likely diagnosis? A) Pituitary insufficiency B) Primary adrenal insufficiency C) Insulin overdose D) Primary hypothyroidism E) Carbon monoxide poisoning

Answer A. Pituitary insufficiency Explanation The problem list for this patient is long: Nightmares in a diabetic patient requiring insulin adjustment Alopecia Hypothermia Delayed reflexes Prerenal state Hyponatremia Hypokalemia TSH and FT 4 both very low FSH and LH both inappropriately low in a postmenopausal female Anion gap = 17 Symptoms and laboratories suggest abnormalities in thyroid, gonadotropin, and adrenal function (nightmares = hypoglycemia). All 3 functions are controlled within the pituitary gland, and any abnormality in the gland (such as an adenoma) can result in defective function of releasing hormones. Therefore, the answer is pituitary insufficiency instead of a primary hormone defect. Insulin overdose and pernicious anemia do not explain the metabolic derangements. Recognize that primary adrenal insufficiency would result in the same hypoglycemic symptoms but would more likely be associated with skin bronzing, hyperkalemia, and hyponatremia due to disruption of the adrenal cortex. These clinical symptoms are absent in cases of secondary adrenal insufficiency.

A patient recently passed a kidney stone in the local emergency department and is referred to your clinic for follow-up. He is a "health freak" and takes megadoses of vitamins and supplements of calcium. As part of your workup, you obtain the following labs: Na+ 140 mEq/L (135-143 mEq/L) K+ 4.0 mEq/L (3.5-5.0 mEq/L) Cl- 104 mEq/L (100-109 mEq/L) HCO3- 24 mEq/L (22-30 mEq/L) Urea nitrogen 12 mg/dL (8-18 mg/dL) Creatinine 1.6 mg/dL (0.6-1.2 mg/dL) Glucose 85 mg/dL (65-110 mg/dL) Ca+2 10.6 mg/dL (8.5-10.5 mg/dL) Phosphate 2.5 mg/dL (2.5-4.5 mg/dL) Albumin 4.6 g/dL (4.0-6.0 g/dL) Vitamin A 120 µg/dL (30-100 µg/dL) 25OH-vitamin D 80 µg/L (10-55 µg/L 1,25(OH)2-vitamin D 68 ng/L (18-62 ng/L) Intact PTH 42 pg/mL (10-65 pg/mL) Which of the following is the most likely cause of his hypercalcemia? A) Primary hyperparathyroidism B) Vitamin D toxicosis C) Unknown malignancy D) Granulomatous disease E) Vitamin A toxicosis

Answer A. Primary hyperparathyroidism Explanation This patient has hypercalcemia and an intact PTH, which is not suppressed. Therefore he has hyperparathyroidism. The PTH should always be suppressed in anyone with hypercalcemia. In this scenario, the PTH is inappropriately normal. His 25OH-vitamin D and vitamin A levels are elevated due to his excessive vitamin intake. Both of these can cause hypercalcemia, but the PTH would be suppressed. The same holds true for granulomatous disease or an unknown malignancy. His 1,25(OH)2-vitamin D may be elevated either by intake of the vitamin or by his primary hyperparathyroidism, but either way it is not the main cause of his hypercalcemia.

A 47-year-old man comes to your clinic for a pre-employment physical. He denies any PMH. His BMI is 31. PE is unremarkable. Labs reveal a fasting plasma glucose of 128 mg/dL. Which of the following is the most appropriate next step in patient care? A) Recheck a fasting blood glucose on a different day. B) Diagnose him with prediabetes. C) Diagnose him with chemical diabetes. D) Diagnose him with diabetes.

Answer A. Recheck a fasting blood glucose on a different day. Explanation This question asks you to demonstrate knowledge of the diagnosis of diabetes mellitus. Recall the following: Criteria for the diagnosis of diabetes mellitus 1. Symptoms of diabetes plus casual plasma glucose concentration ≥ 200 mg/dL (11.1 mmol/L). Casual is defined as any time of day without regard to time since the last meal. The classic symptoms of diabetes include polyuria, polydipsia, and unexplained weight loss. or 2. FPG ≥ 126 mg/dL (7.0 mmol/L). Fasting is defined as no caloric intake for at least 8 hr. or 3. 2-hr postload glucose ≥ 200 mg/dL (11.1 mmol/L) during an OGTT. The test should be performed as described by WHO, using a glucose load containing the equivalent of 75 gm anhydrous glucose dissolved in water. or 4. Hemoglobin A1c of ≥ 6.5% In the absence of unequivocal hyperglycemia, these criteria should be confirmed by repeat testing on a different day. You should use the same test the 2 nd time and not switch to a different test. The OGTT is not recommended for routine clinical use.

A woman with Type 1 diabetes comes to your office. She has been followed by another physician but hasn't been feeling right and wants to try a different physician. She heard that you are the best internist in town. She is 50 years old and has had diabetes for 35 years. She has retinopathy (being followed by an ophthalmologist) and mild microalbuminuria. Her medications include NPH insulin 15 units every morning and 60 units at bedtime; a rapid-acting insulin analogue 6-12 units with each meal, which she adjusts depending on the pre-meal capillary glucose level; an ACE inhibitor every evening; and aspirin 81 mg daily. Her glucometer log shows glucose levels generally okay except for the morning fasting glucose, which is elevated. The physical examination is unremarkable except for some retinal microaneurysms. She has her most recent laboratory tests from last week, which are generally okay. The HbA1c is 5.8%. She is very proud of her HbA1c and reports that her other physician had been increasing the evening NPH in an attempt to lower the morning fasting glucose, which has been steadily high despite increasing the evening insulin. She reports to you that she has been experiencing terrible, vivid nightmares during the night and often wakes up in a cold sweat, but attributes this to a scary movie that she saw 2 months ago. Which of the following is the most appropriate action to take at this time? A) Reduce her evening NPH insulin and follow closely. B) Begin an anxiolytic to help her nightmares. C) Add an insulin sensitizer to reduce her insulin needs. D) Increase the evening NPH insulin again to reduce the morning fasting glucose levels. E) Switch her to 70/30 insulin mix twice daily.

Answer A. Reduce her evening NPH insulin and follow closely. Explanation Her symptoms of vivid nightmares and waking in a cold sweat are compatible with nocturnal hypoglycemia. The most likely explanation for the early morning hyperglycemia is that the nocturnal hypoglycemia is causing the contra-insular hormones (glucagon, cortisol, growth hormone, and epinephrine) to be secreted. Many Type 1 diabetics lose the ability to secrete glucagon in response to hypoglycemia, but the remaining hormones are unaffected. These hormones will raise the glucose level. However, insulin-deficient persons such as this patient do not have the ability to make their own insulin to control the hyperglycemic effect of the contra-insular hormones, and her morning fasting glucose levels are elevated. Every patient taking insulin needs to perform self-monitoring of their blood glucose levels. They should check their glucose before and 2 hours after every meal, at bedtime, and in the middle of the night around 3 a.m. Of course it's impractical to constantly check glucose levels this often on a daily basis. One alternative is to have patients check their glucose level twice every day, choosing different times each day so that the log book will have some numbers at each time point. The 3 a.m. glucose level should be checked at least once each month, but doesn't need to be checked much more frequently—unless the patient is having evidence of nocturnal hypoglycemia or has changed the insulin regimen. Once this patient no longer has hypoglycemia, the insulin can always be carefully and slowly increased. It is inappropriate to begin an anxiolytic rather than decreasing the insulin. Hypoglycemia can be life-threatening. Increasing her evening insulin further could be fatal. There is no evidence that she has insulin resistance in addition to her insulin deficiency, and adding an insulin sensitizer is inappropriate. Switching her to a 70/30 premix is also inappropriate. The goal of treating a Type 1 diabetic is to replace the insulin in a physiologic way. Her insulin regimen is already physiologic by offering basal coverage with NPH twice daily and bolus coverage with a rapid-acting insulin analog with each meal.

A 62-year-old man comes to see you because of a lump in his throat that has been there for about 1 year. You palpate a 2-cm firm nodule in the left lower lobe of his thyroid. The mass is nontender and movable. The remainder of his thyroid is non-palpable, and he has no palpable lymph nodes in his neck. You find him to be clinically euthyroid. You order thyroid tests and have him return to your clinic in 1 week. The nodule has not changed, and his TSH and FT4 are both within normal limits. Which of the following is the next best step in patient care? A) Refer him for a thyroid fine-needle biopsy and aspiration (FNA). B) Begin thyroxine therapy in the hope that the nodule may shrink. C) Refer him to a surgeon for a hemithyroidectomy. D) Follow him for several years, watching for any changes in the nodule. E) Refer him to a surgeon for a near-total thyroidectomy.

Answer A. Refer him for a thyroid fine-needle biopsy and aspiration (FNA). Explanation This patient is euthyroid, but with a thyroid nodule. Perform a thyroid FNA yourself, or refer him to someone who can. If the biopsy is not conclusive, then most would repeat it. If the lesion is hot and the patient is euthyroid, they can either be followed or, if the patient is hyperthyroid, radioiodine therapy or surgery can be recommended. A near-total thyroidectomy would be needed only if cancer was present. Many physicians in the past have tried to shrink nodules with thyroxine, thinking that exogenous thyroxine will lower the TSH, which will reduce the growth stimulation to the nodule. Unfortunately, studies have shown this not to be the case; thyroxine should not be given in the hope of shrinking a nodule. Considering how easy and safe a thyroid biopsy is, it would be inappropriate to just follow him. Though the vast majority of nodules are benign, cancer cannot be excluded by simply watching the nodule.

A new patient comes to your office complaining of a lump in her neck. She is 18 years old and reports first noticing the lump 6 months ago. She delayed seeing you because she thought it would go away. She reports no change in size during the past 6 months and denies any problems with swallowing or breathing. There is no family history of thyroid disease, and she denies any exposure to radiation. Her voice is normal and the nodule is 3 cm, nontender, and movable. She does not exhibit any cervical lymphadenopathy. She is clinically euthyroid. You tell her that thyroid nodules are fairly common and thyroid cancer is relatively rare, but that you wish to begin a workup. She asks you if she has any increased risk for thyroid cancer. Which of the following should be your response? A) She has an increased risk because of her age. B) She has an increased risk because of her gender. C) You must first check her TSH level. D) She has no increased risk. E) You must first check her free T4 level.

Answer A. She has an increased risk because of her age. Explanation This question requires you to know clinical features suggesting a high risk for malignancy in a thyroid nodule. The characteristics indicating an increased risk for thyroid malignancy are as follows: Family history of thyroid cancer Exposure of the neck to radiation that may be therapeutic (e.g., radiation therapy to the neck for lymphoma, etc.); intentional but misguided radiation (e.g., radiation to shrink the thymus in a neonate, as was the practice in the 1950s); nuclear fallout (thyroid cancer is a major problem in the regions affected by Chernobyl) Pain in the anterior neck Dysphagia Symptoms of respiratory obstruction New nodule in someone < 20 or > 60 years of age Vocal cord paralysis (hoarseness) Thyroid nodule that is firm, fixed, and/or growing Thyroid nodule accompanied by regional cervical lymphadenopathy or Horner syndrome The patient in this scenario is under 20 years of age, which incurs an increased risk for malignancy.

A new patient comes to your clinic for routine follow-up. He is 41 years old and has Type 2 diabetes. He reports fairly good compliance with diet and exercise. His BMI is 24.8 kg/m2, and he briskly walks 3 miles every day after work. His last dilated eye exam was 6 months ago and no abnormalities were reported. He checks his capillary glucose levels once or twice a day, and his logbook shows reasonable glycemic control. He has a history of hyperlipidemia and is fully compliant with the diet recommended by a dietitian. He has tried and failed to tolerate niacin, bile acid sequestrants, and fibrates because of side effects. He also experienced hypersensitivity to ezetimibe. Your exam today is unremarkable, and his feet are okay. You order repeat fasting lipids and HbA1c. He comes back for follow-up of his lab tests, which are reported as follows: HbA1c 6.9 % ALT 120 IU/L (0-55 IU/L) LDL 175 mg/dL HDL 33 mg/dL Triglycerides 127 mg/dL He denies any alcohol use. How should you respond to his dyslipidemia? A) Start a moderate intensity statin despite elevated ALT. B) Further restrict his diet and recheck. C) Recheck ALT every 6 months and start a statin once ALT is normal. D) Encourage more exercise and recheck. E) Determine his LDL particle size.

Answer A. Start a moderate intensity statin despite elevated ALT. Explanation This question tests your understanding of the dangers of dyslipidemia and the risk of using a statin in the presence of mildly elevated ALT. Of course, one would normally avoid a statin in the presence of elevated hepatic transaminases. However, this patient has a markedly elevated risk for cardiovascular disease and absolutely needs treatment. He is on an appropriate diet and exercises, so no additional lifestyle recommendations are likely to provide much additional help. Unfortunately, he is not able to take the drugs that are often used to treat hyperlipidemia. However, he really needs treatment. In this setting, it is appropriate to use a statin despite the mildly elevated ALT. Begin a statin at the lowest approved dose and slowly titrate the dose while monitoring his hepatic function very carefully.

A 27-year-old woman was found to have a 4-mm prolactin-secreting pituitary tumor 1 year ago. Her prolactin was 195 ng/mL (< 20), and she was started on 5 mg of bromocriptine daily. Today, she reports a positive pregnancy test. What advice should you give with respect to the bromocriptine and follow-up during pregnancy? A) Stop bromocriptine; continue regular follow-up with an obstetrician. B) Repeat prolactin level monthly until delivery. C) Continue bromocriptine. D) Schedule MRI and formal visual field testing. E) Stop bromocriptine; switch to an equivalent dose of cabergoline.

Answer A. Stop bromocriptine; continue regular follow-up with an obstetrician. Explanation This woman has a prolactin-secreting microadenoma, and the likelihood of tumor growth with small tumors during pregnancy is < 2%. There is no reason to switch to cabergoline, because she has already conceived. Prolactin levels increase over the course of pregnancy, as the breasts are prepared for lactation, so there is no rationale to check PRL levels during gestation or continue bromocriptine. Because so few women with microadenomas have tumor growth during pregnancy, it is not necessary to do an MRI or visual field examination unless there are clinical signs of tumor growth (headache, visual loss, etc.).

A patient with a severe asthma attack was treated with prednisone 50 mg daily for 5 days. The pulmonologist says the patient no longer needs steroids and signs off on the case. Which of the following steps should you take now? A) Stop the prednisone abruptly. B) An insulin tolerance test to assess hypothalamic function. C) ACTH stimulation test to assess adrenal function. D) CRH stimulation test to assess pituitary function. E) Switch to 5 mg/day and periodically check for return of adrenal function.

Answer A. Stop the prednisone abruptly. Explanation A week or less of high-dose glucocorticoids would not be expected to suppress the hypothalamic-pituitary-adrenal axis to the point that steroids need to be continued while waiting for return of function. Two weeks of high-dose steroids would certainly suppress the axis and require continuation of a replacement dose while checking for return of function. The period between 1 week and 2 weeks is very variable, and replacement dose should be continued to err on the side of caution.

A 26-year-old female presents for evaluation of neck pain and fatigue. She described the pain as located in the front of her neck and does not radiate, but has been constant over the last week. She reports she recently had an upper respiratory infection with cough, nasal congestion, sore throat, and fever last month. Review of systems elicits that the patient has felt fatigue over the last 2-3 weeks as well. On physical exam, her HR is 95, and her BP is 145/87. HEENT exam reveals a clear oropharynx and nares. Thyroid exam reveals a smooth, enlarged thyroid gland that is exquisitely tender to palpation. The remainder of the physical exam is unremarkable. Thyroid function testing reveals: TSH - < 0.01 uU/mL (0.5-5.0 uU/mL) Free T 4 - 2.6 ng/dL (0.9-2.4 ng/dL) Radioactive iodine uptake scan reveals decreased uptake throughout the thyroid gland. Which of the following is the most likely diagnosis? A) Subacute thyroiditis B) Toxic multinodular goiter C) Graves disease D) Medullary thyroid carcinoma E) Chronic lymphocytic thyroiditis

Answer A. Subacute thyroiditis Explanation This patient has subacute thyroiditis. Most forms of thyroiditis will present with hyperthyroid symptoms initially due to release of hormones from cell destruction in the gland, followed by a euthyroid phase that can then sometimes progress to hypothyroidism. This occurs in both chronic lymphocytic thyroiditis and subacute thyroiditis. Subacute thyroiditis typically occurs after an antecedent trigger like a viral infection and often produces a tender thyroid. A tender thyroid gland is a common finding with subacute thyroiditis that would not be expected in chronic lymphocytic thyroiditis. Toxic multinodular goiter and Graves disease are additional causes of hyperthyroidism. With both, you would expect to see normal to likely increased uptake on a radioactive iodine uptake scan, indicating increased synthesis of thyroid hormone. Both would also more likely present with complaints of symptoms related to hyperthyroidism rather than neck pain. Medullary thyroid carcinoma produces calcitonin, not thyroid hormone, and typically grows as a solitary nodule that then metastasizes to surrounding lymph nodes.

A 21-year-old woman comes to see you in the university health clinic complaining of no menses for two months. She has been doing well in her studies and denies any unusual stress. On the contrary, she greatly enjoys the swim team and has been doing quite well lately. She consumes 2500 kcal/day. She reports that her menses began at age 13, and her periods have been regular as far back as she can remember. Her vital signs are BP 100/70 mmHg and pulse 65/min. Her BMI is 24 kg/m2 and she has no documented weight loss over the last six months. Your examination reveals a well-developed female with no abnormalities that can be detected by your pelvic exam. She has mild acne, no clitoromegaly, her breasts are at Tanner stage V, her pubic hair pattern is at Tanner stage IV, and nothing can be expressed from her breasts. You notice that she walks with a slight limp, which she attributes to an old ankle injury due to a boating accident and says it bothers her only during weight-bearing activities. A urine pregnancy test is negative, and she denies sexual activity since she broke up with her boyfriend 9 months ago. She was depressed at the time, but filled the void with more time in the pool. She denies any prescription or illicit drug use. Which of the following causes of amenorrhea can be eliminated from the differential diagnosis? A) Swimming-induced amenorrhea B) A pituitary tumor C) Polycystic ovarian syndrome D) Exogenous androgens E) Congenital adrenal hyperplasia

Answer A. Swimming-induced amenorrhea Explanation This question requires you to know some of the many causes of secondary amenorrhea and what type of exercise does not cause amenorrhea. This is a young, physically active woman who stopped having her usual periods. Many things can cause secondary amenorrhea, including all incorrect choices mentioned in the list of possible answers. However, it is important to note that exercise-induced amenorrhea is typically accompanied by a calorie deficit low BMI. In addition, swimmers in general are less likely to develop secondary amenorrhea compared to runners and dancers. Don't forget the most common cause of secondary amenorrhea is pregnancy! Over 60% of women with secondary amenorrhea have hyperandrogenism. PCOS is one of the most common endocrine disorders and is a diagnosis of exclusion. 2 out of 3 criteria must be met: (1) hyperandrogenism (2) oligomenorrhea/amenorrhea (3) polycstic ovaries on ultrasound. Given the patient's amenorrhea and acne, she should also undergo work up for an androgen-secreting tumor and late-onset congenital adrenal hyperplasia. Elevated LH and FSH in the setting of secondary amenorrhea means that the patient has loss of negative feedback due to premature ovarian failure (POF). Decreased FSH and LH in the amenorrheic woman tell you that the pituitary is not making the hormones, either because the pituitary itself or the hypothalamus is diseased or because of excessive negative feedback from another source such as exogenous steroids. This state is called "hypogonadotropic hypogonadism,"

A 50-year-old male has a 1-month history of tremor, palpitations, heat intolerance, and hyperdefecation. He has no history of thyroid disease, but his sister takes thyroid hormone. His pulse is 100; his thyroid is tender to palpation and mildly enlarged and non-nodular. He has no proptosis. A free T 4 is 2.1 (0.9-1.5) and TSH is 0.01 (0.2-4.2). A serum thyroglobulin level is elevated. What is the most likely explanation for the elevated free T 4 ? A) Graves disease B) Subacute thyroiditis C) Surreptitious use of thyroid hormone D) Toxic multinodular goiter

Answer B. Subacute thyroiditis Explanation The important features are the short duration of symptoms, lack of proptosis, and the tender gland. Graves disease would have an enlarged, smooth, nonpainful gland. No nodules are palpable, as would be expected in a multinodular goiter. Surreptitious use of T 4 would not lead to a painful gland and would not be associated with elevated serum thyroglobulin.

Your favorite patient comes to your clinic. He is 45 years old and has a recently diagnosed 5-mm prolactinoma, which you started treating with low-dose bromocriptine. He complains to you about constant nausea and vomiting since starting bromocriptine. He retches several times in your presence. What is the next best thing to do? A) Switch him to cabergoline. B) Try some prochlorperazine or other anti-nausea drug. C) Add cabergoline to bromocriptine. D) Stop the bromocriptine and follow his prolactinoma with serial MRIs. E) Refer him to a neurosurgeon for resection of the prolactinoma

Answer A. Switch him to cabergoline. Explanation The question states that the patient has a prolactinoma and is being treated with bromocriptine. Unfortunately, he is experiencing considerable nausea and vomiting, which are common side effects of bromocriptine; many patients are unable to tolerate it. An alternative drug, cabergoline, is available. This drug is just as effective at reducing prolactin levels and is much less likely to cause nausea and vomiting. Unfortunately, cabergoline is expensive. In some centers, bromocriptine is the initial drug of choice because it is cost-effective. However, cabergoline has become the drug of choice in many centers because it has fewer side effects and is much better tolerated. It would be inappropriate to just stop the bromocriptine without starting the cabergoline, because without a brief period of overlap, the patient would most likely experience a flare-up in his symptoms. Resection of a prolactinoma should be done only when the mass effect of a large tumor warrants it, such as seizures or an impaired field of vision. Adding prochlorperazine for nausea is not appropriate, because he hasn't tried a course of cabergoline. Finally, the addition of cabergoline to bromocriptine should not be done. These two drugs work by the same mechanism, and using them together would not lessen the severity of the nausea and vomiting and would increase any potential side effects.

A patient of yours is a 35-year-old man who had a thyroid nodule that proved to be cancerous. After his thyroidectomy, the final pathology report mentioned a 4.2-cm mass containing a well-differentiated papillary cancer. There was no growth of the cancer into adjacent structures and no evidence of metastases into any lymph nodes. He received high-dose radioactive iodine ablation after surgery. His total body scan was negative, and he is now on a suppressive dose of thyroxine. He is in your office for a routine follow-up. He is clinically euthyroid, the TSH is 0.30 mU/L (0.5-5.0 mU/L), and the free-thyroxine (FT4) is 1.4 ng/dL (0.7-1.5 ng/dL). He asks you about the risk of recurrence of his thyroid cancer. Which of the following responses should you give him? A) The risk for recurrence is increased due to the size of the tumor. B) The risk for recurrence is increased due to the type of cancer. C) The risk for recurrence is increased due to his age. D) The risk for recurrence is increased due to his gender. E) There is no increased risk for recurrence.

Answer A. The risk for recurrence is increased due to the size of the tumor. Explanation This question requires you to know the risk factors for recurrence of thyroid cancer. Poor prognostic factors and risk factors for cancer recurrence are as follows: Tumor size > 4 cm Age > 40 years at diagnosis Direct invasion of local structures Distant metastases Poor tumor differentiation or advanced tumor grade Mediastinal or bilateral cervical lymph node metastases (if papillary cancer) Extensive capsular and vascular invasion (if follicular cancer) The patient has an increased risk for cancer recurrence due to the size of the tumor being greater than 4 cm. Gender is not a factor; his age at the time of diagnosis was less than 40; and original complaints do not increase the risk of recurrence. He is on an appropriate dose of thyroxine, and his TSH level is within the target range to decrease the risk of recurrence. Nevertheless, he still has an elevated risk due to tumor size.

A 40-year-old man is referred to you after a DXA scan done at a health fair screening indicated low bone density with a T-score of -1.6. He tries very hard to stay healthy by exercising and taking mega-doses of vitamins. On examination, nothing remarkable is found. Chem-7, LFTs, and albumin are normal. Patient's Normal Calcium 10.7 mg/dL 8.5-10.5 Phosphorus 4.7 mg/dL 1.5-4.5 25-OH2-D 60 microgram/L 9-52 1,25-OH2-D 13 ng/L 16-60 iPTH 5 pg/mL 18-73 Testosterone 550 ng/dL 260-1,000 Which of the following is the most likely cause of his osteopenia and elevated Ca? A) Vitamin A intoxication B) Surreptitious use of anabolic steroids C) Excess calcium in his diet D) Primary hypoparathyroidism E) Vitamin D

Answer A. Vitamin A intoxication Explanation His 25-OH-D levels may be mildly elevated from excess supplementation, but the body regulates the active 1,25-OH2-D, which is low due to low iPTH. The real culprit may be vitamin A intoxication, which increases intestinal absorption of calcium and stimulates osteoclasts to release calcium and phosphorus from bone. The elevated calcium suppresses PTH, and the reduced PTH results in less urinary clearance of phosphorus.

A 65-year-old man has had benign prostatic hypertrophy for several years. He is still able to urinate effectively, but his symptoms have started to worsen. His urologist has scheduled a TURP for next week and referred him to you for a medical preoperative evaluation. Besides the expected prostate enlargement, your examination shows mild tachycardia and atrial fibrillation. Suspecting hyperthyroidism, you order thyroid function tests. The results are TSH 0.1 μIU/mL (normal: 0.3-5.0 μIU/mL) and FT4 2.5 ng/dL (normal: 0.7-1.5 ng/dL). Which of the following is the best advice for you to give the urologist? A) Give an ablative dose of radioactive iodine immediately before the operation. B) Begin antithyroid medication now and postpone the surgery until he is euthyroid or nearly so. C) Proceed with the operation and begin antithyroid medication afterward. D) Postpone the surgery and ask the urologist to refer the patient to a cardiologist for treatment of his atrial fibrillation. E) Recheck his thyroid function postoperatively.

Answer B. Begin antithyroid medication now and postpone the surgery until he is euthyroid or nearly so. Explanation This question requires you to understand the dangers of hyperthyroidism in a patient going into surgery. This patient has signs of hyperthyroidism, which is verified by laboratory tests. In a patient with hyperthyroidism, any kind of surgery can trigger a severe uncompensated hyperthyroidism (thyroid storm), which can be fatal. Treatment of the hyperthyroidism should be initiated before the surgery takes place. The question of whether to postpone the operation is a little more problematic and depends on whether the surgery is emergent, urgent, or routine. Obviously, if the surgery is an emergency and cannot be postponed, ask the anesthesiologist to administer iodine to inhibit thyroid hormone synthesis (this can be administered as intravenous radiocontrast agents), β-blockers to block the adrenergic effects of hyperthyroidism, and glucocorticoids to protect from the stress of hyperthyroidism and to inhibit the conversion of T4 to T3. If the surgery is routine and can be delayed, as in this case, the patient should be started on antithyroid medications, and the operation should be postponed until the patient is rendered euthyroid or nearly euthyroid. An operation that is urgent, but not emergent, can be postponed for a limited amount of time. During this interval, begin antithyroid medications (PTU or methimazole), iodide, and β-blockers. Though the patient will probably not be euthyroid before surgery, his hyperthyroidism will at least be lessened in severity and may protect him from developing thyroid storm.

A 20-year-old female is referred for treatment of facial hair, which has been present since age 12. Her menstrual periods are regular; her BMI is 20. She has been taking an oral contraceptive for two years. She has dark hair on the upper lip, cheeks, and areolae, and her pelvic exam is normal. Her A1c is 5.1%, and her testosterone level is 20 (6-82). What should you recommend now? A) Transvaginal ultrasound. B) Begin spironolactone. C) Measure 17-hydroxyprogesterone. D) Begin metformin.

Answer B. Begin spironolactone. Explanation This is an example of idiopathic hirsutism. It is not a clinical picture of 21-hydroxylase deficiency, so measuring 17-hydroxyprogesterone is not appropriate. The symptoms are not right for PCOS as she has regular menses, is thin, and has a normal A1c. Her complaint is hirsutism, and the best treatment to slow hair growth is spironolactone, which is an aldosterone and androgen receptor antagonist. An oral contraceptive will also help slow hair growth but she is already taking it. Importantly, spironolactone should never be given without a form of contraception as it is a teratogen and can affect sex differentiation during embryogenesis.

A psychiatrist in your community refers a patient to you who is an 80-year-old woman being treated for depression. She reports generalized weakness, fatigue, dry skin, weight gain, and constipation. Her past medical history includes CHF and stable angina. Your examination reveals periorbital edema, skin that is cool and dry, loss of the lateral third of her eyebrows, mild bradycardia, and slow relaxation phase of her deep tendon reflexes. You strongly suspect hypothyroidism and check TSH and FT4. The TSH is 95 μIU/mL (normal: 0.3-5.0 μIU/mL), and the FT4 is 0.1 ng/dL (normal: 0.7-1.5 ng/dL). She obviously has severe hypothyroidism. Which of the following should you do next? A) Administer thyroxine 500 μg IV and triiodothyronine 20 μg IV qd x 3. B) Begin thyroxine 25 μg PO qd. C) Begin thyroxine 300 μg PO qd. D) Begin thyroxine 150 μg PO qd. E) Administer thyroxine 500 μg IV every day for 5 doses.

Answer B. Begin thyroxine 25 μg PO qd. Explanation This patient has severe hypothyroidism along with poor cardiac function. Though hypothyroidism impairs cardiac function, it also decreases the demand being placed on the heart. Thus the heart is not always working as hard in hypothyroidism as it may be working in someone who is euthyroid. There is no question that her hypothyroidism needs to be addressed, but you are being asked to choose which treatment is most appropriate. Intravenous thyroxine will rapidly correct her hypothyroidism, especially with the addition of triiodothyronine, but this will also rapidly increase the demands placed on her heart. In the face of her impaired cardiac function, this sudden increased demand could easily exceed her cardiac capacity and have dire consequences. The remaining three choices are all oral thyroxine at different strengths. The most appropriate choice in this case is the 25 μg dose. This dose will not have much effect on her hypothyroidism, so it will not greatly increase her cardiac work. The dose can be steadily increased every 3-6 weeks if she tolerates it. The best course of action is to start low and go slow. This will give her heart a chance to increase its conditioning and be able to tolerate the next dose increase

A 56-year-old woman comes to see you for a routine Pap smear and general checkup. She reports mild fatigue but continues with a very active lifestyle, including tennis one day a week and a 2-mile brisk walk 5 days per week. Her past medical history is unremarkable, she doesn't smoke, and she is taking over-the-counter calcium supplements. The physical exam is unremarkable, and the Pap smear is done without difficulty. You tell her that she will probably dance on your grave. You order some blood tests and tell her to call for the results next week. The Pap smear is normal. Laboratory: Na+ 140 mEq/L (135-143 mEq/L) K+ 4.0 mEq/L (3.5-5.0 mEq/L) Cl- 104 mEq/L (100-109 mEq/L) HCO3- 24 mEq/L (22-30 mEq/L) Urea nitrogen 9 mg/dL (8-18 mg/dL) Creatinine 0.8 mg/dL (0.6-1.2 mg/dL) Glucose 92 mg/dL (65-110 mg/dL) Ca+2 9.7 mg/dL (8.5-10.5 mg/dL) FT4 0.9 ng/dL (0.7-1.5 ng/dL) TSH 10.6 mU/L (0.5-5.0 mU/L) Anti-thyroid antibodies Positive Fasting lipids: LDL 175 mg/dL HDL 41 mg/dL TG 107 mg/dL Which of the following is the best plan of action at this time? A) Order a pelvic ultrasound. B) Begin thyroxine 50 µg per day and check lipids, TSH, FT4, and anti-thyroid antibodies in 2-3 months. C) Schedule a revisit appointment in one year. D) Start a statin. E) Order a 75-gram oral glucose tolerance test (OGTT) to rule out diabetes.

Answer B. Begin thyroxine 50 µg per day and check lipids, TSH, FT4, and anti-thyroid antibodies in 2-3 months. Explanation This patient may very well have subclinical hypothyroidism. By definition, the diagnosis is very difficult and easily missed: The FT4 is normal and the TSH may be normal or minimally elevated. This diagnosis is important because of the implications. Positive anti-thyroid antibodies contribute to the diagnosis of subclinical hypothyroidism and increase the risk of transforming into overt hypothyroidism in the future. Patients with subclinical hypothyroidism and a TSH>10 mU/L should be treated regardless of symptoms. For TSH between 5-10 mU/L, patients should be treated if they have symptoms of hypothyroidism. The Rotterdam study clearly showed that women ≥ 55 years of age with subclinical hypothyroidism have a significantly elevated risk of myocardial infarction and aortic atherosclerosis. Nevertheless, a clear benefit from treatment has not yet been published. However, hypothyroidism is well known to cause an elevated LDL, and her LDL is 175 mg/dL. Even though she is not symptomatic for hypothyroidism, her elevated TSH >10 mU/L in conjunction with her elevated LDL are both factors in your decision to treat with thyroxine supplementation. It is premature to begin statin drug therapy, since treatment of hypothyroidism often reduces LDL levels. Starting a low-dose trial of thyroxine is very appropriate in this case. Since her anti-thyroid antibodies (anti-TPO) are positive, I would continue her thyroxine therapy even if the LDL does not improve. If the LDL remains elevated, you should consider statin therapy. A year-long revisit is too prolonged a follow-up to address both the lipid disorder and subclinical hypothyroidism. And finally, her likelihood of having an ovarian or uterine cancer in this post-menopausal patient without symptoms or an abnormal pelvic exam is very low. Therefore, the indication for a pelvic ultrasound is unnecessary.

A 48-year-old woman comes to your clinic with fatigue and galactorrhea. Thin white fluid can be expressed from each breast. She has no breast masses or tenderness. Her skin is dry. Recent mammogram was normal. Prolactin level: 40 (1.4-14.2) MRI: 8-mm pituitary mass Which of the following is the next best step in management? A) Start bromocriptine. B) Check TSH. C) Look for an unknown nonpituitary cancer. D) Refer her to a neurosurgeon for resection of a prolactinoma. E) Recheck prolactin level in 6-12 months.

Answer B. Check TSH. Explanation This question asks you to navigate the differential diagnosis of galactorrhea. Remember that hypothyroidism can raise prolactin levels, so exclude thyroid disease with a TSH level before definitively diagnosing a prolactinoma. Also recall, prolactin levels > 200 correlate with adenomas (prolactinomas). Observation is not appropriate in cases of galactorrhea.

A 43-year-old woman presents to your office for a preventive health visit. Her past medical history is significant for chronic kidney disease due to focal glomerulosclerosis with an estimated GFR of 20 mL/min/1.73 m2. She currently takes furosemide, amlodipine, metoprolol, ferrous sulfate, and erythropoietin. She reports being compliant with her current medications. On physical exam, blood pressure is 138/85, heart rate 75/min. Examination reveals 1+ pitting edema of the lower extremities. The remainder of the physical exam is unremarkable. Review of laboratory studies collected by her nephrologist 3 months ago reveal: Hemoglobin A1c: 5.4% Fasting plasma glucose: 130 mg/dL 2-hour oral glucose tolerance test: 175 mg/dL Which of the following tests would be recommended to screen this patient for diabetes? A) Hemoglobin A1c B) Fasting plasma glucose C) No additional testing needed D) 2-hour oral glucose tolerance test E) Serum fructosamine

Answer B. Fasting plasma glucose Explanation There have been numerous recommendations published on screening for diabetes. There are currently 3 validated tests: hemoglobin A1c, fasting plasma glucose, and an oral glucose tolerance test. Hemoglobin A1c testing is now widely used in screening for diabetes. However, hemoglobin A1c testing is not valid in patients with abnormal red cell turnover, such as those with hemolytic anemia, pregnancy, blood loss/transfusions, or use of erythropoietin. Since this patient is on erythropoietin, recommended screening would use either a fasting plasma glucose or an oral glucose tolerance test. This patient underwent screening by her nephrologist with equivocal results. The current recommendation for screening and diagnosis of diabetes would be to repeat the test that indicates a diagnosis of diabetes. Since her 2-hour oral glucose tolerance test was negative, it would not be recommended to repeat this time to screen for diabetes. For this patient, the fasting plasma glucose was > 126 mg/dL, indicating possible diabetes. If the FPG remains > 126 mg/dL on repeat testing, the diagnosis of diabetes would be confirmed. Serum fructosamine has been used as a test to monitor long-term glycemic control in patients in whom an A1c measurement would be invalid, but is not validated as a screening method to diagnose diabetes

Which of the following thyroid disorders will cause a high nuclear medicine thyroid uptake? A) Amiodarone B) Graves disease C) Iodine excess D) Postpartum thyroiditis E) Subacute thyroiditis

Answer B. Graves disease Explanation Graves will cause a high uptake. Any thyroiditis may cause a low uptake because a "sick" thyroid gland may not be functioning properly. Excess iodine will suppress the thyroid uptake because (1) the gland is already saturated with iodine and cannot uptake much more, and (2) the non-radioactive iodine competes with the radioactive iodine so the gland is more likely to take up non-radioactive iodine than radioactive iodine. Amiodarone can suppress thyroid uptake for two reasons. It can cause a thyroiditis, and it provides a very substantial iodine load. Iodine deficiency leads to an increased thyroid uptake because the gland is deficient of iodine and will avidly take up the radioactive iodine.

A mother brings her 18-year-old son to see you because his science teacher thinks something may be wrong. The class was learning to identify Barr bodies on buccal smears when it was noticed that the young man was the only male in the class with a Barr body. His mother says that he is timid and doesn't do well in school. Lately, the other boys have been making fun of him. His mother reports that he had the mumps at age 16. After his mother leaves the room, you ask him about erections and masturbation. He denies them both. You examine him and find that his arm span is 70 inches and his height is 69 inches. His testes are very small (< 1 in) and quite firm. He has mild bilateral gynecomastia and sparse facial hair. He reports that he hasn't started shaving yet. You order labs and have him come back to see you in 2 weeks. The following are the laboratory tests: Chem-7 WNL CBC WNL Testosterone low for age and gender FSH high for age and gender LH high for age and gender Which of the following do you tell him? A) He has complete testicular feminization. B) He has Klinefelter syndrome and should start testosterone, 100 mg IM weekly. C) He has hypogonadotropic hypogonadism. D) He has Kallmann syndrome and should start testosterone, 100 mg IM weekly. E) He suffers from primary hypogonadism due to postpubertal mumps infection.

Answer B. He has Klinefelter syndrome and should start testosterone, 100 mg IM weekly. Explanation This question tests your knowledge of Klinefelter syndrome and other causes of hypogonadism. He has Klinefelter syndrome, which is characterized by very small and fibrotic testes. The genotype is 47,XXY, and hence a Barr body (condensed chromatin seen in female cells due to an inactive X chromosome) can be seen in these men. They often have gynecomastia and poor social skills. They are not able to produce normal amounts of testosterone. The treatment is testosterone replacement, which is often done with weekly injections. One important point to remember is that the arm span is normal (arm span - height < 5 cm or 2½ inches). He does not have Kallmann syndrome because such patients have an increased arm span (arm span - height > 5 cm or 2½ inches); testes that are larger than seen in Klinefelter syndrome, although they are still small; a normal genotype (46,XY) and no Barr bodies; usually no gynecomastia; have adequate social skills; and have an impaired sense of smell. Primary hypogonadism due to a postpubertal mumps infection would result in small soft testes, but they will be larger than seen in either Klinefelter or Kallmann syndrome. He clearly has primary hypogonadism and cannot have hypogonadotropic hypogonadism because the gonadotropic hormones LH and FSH are elevated, trying to compensate for the primary defect. He also does not have complete testicular feminization because these 46,XY individuals develop as phenotypical women but without a uterus or full introitus.

A 41-year-old woman is evaluated in the emergency department for headaches, which have been worsening over the past 6 months. She takes no medications and has no past medical history. She denies fatigue or facial flushing. She has not lost weight but has noted palpitations. She has no family history of hypertension. BP is 188/106 supine and 156/92 after standing. HR is 100 without change. Her thyroid is of normal size, cardiac exam is normal, and she has no pedal edema. Laboratory: Serum sodium 135 mEq/L (135-145 mEq/L) Serum potassium 3.9 mEq/L (3.5-5.3 mEq/L) Serum glucose (fasting) 139 mg/dL (80-126 mg/dL) Which step is best for her subsequent management? A) Measure serum cortisol at 8 a.m. after 1 mg dexamethasone at midnight. B) Measure 24-hour urine fractionated metanephrines. C) Measure plasma fractionated catecholamines. D) Measure serum aldosterone and plasma renin activity. E) Perform CT of the abdomen with IV contrast.

Answer B. Measure 24-hour urine fractionated metanephrines. Explanation Key points: Headache is the most common presenting symptom of patients with pheochromocytoma, and orthostasis is a common physical finding. 24-hour urine metanephrines is the best first test to use for screening for pheochromocytoma. Although hyperaldosteronism can cause hypertension and hypokalemia, it is not associated with orthostasis. Serum potassium is in the 3.5-4.0 range in 25% of patients with hyperaldosteronism, but serum sodium is usually in the upper range of normal. The overnight dexamethasone test is an effective screen for Cushing syndrome, but no physical findings are present to suggest this diagnosis. Plasma catecholamines are not a practical screening test for pheochromocytoma because the blood sample needs to be drawn from an indwelling IV in a recumbent patient. Otherwise, spurious elevations are common. Syndromes of adrenal hypertension should always be diagnosed endocrinologically first, before performing CT scans, because of the common occurrence of non-functioning adrenal incidentalomas.

A 60-year-old has fatigue, weight loss, abdominal pain, and loss of axillary hair. His cortisol is 1.2 and stimulated to 5 after Cortrosyn®. You start 5 mg of prednisone. However, his symptoms do not improve, and he has cold intolerance and constipation. His TSH is 0.9 (nl). What should you do? A) Add fludrocortisone. B) Measure free T 4 . C) Increase prednisone to 10 mg daily. D) Measure testosterone.

Answer B. Measure free T 4 . Explanation He has clinical and biochemical signs of secondary adrenal insufficiency along with symptoms of hypothyroidism. The reason his TSH is not elevated is that he has secondary hypothyroidism (↓ FT 4 , ↓ TSH). Increasing prednisone or adding fludrocortisone is not necessary, and low testosterone will not explain his symptoms. It is always important to assess thyroid function in the presence of cortisol deficiency.

A 75-year-old man presents with visual loss. He has cold intolerance, fatigue, weight gain, and headaches. His exam shows BP 110/60, pale skin, ↓ beard growth, and breast enlargement. Lab shows prolactin 24, cortisol 2 (↓), testosterone 50 (↓), and free T4 0.4 (↓). Replacement therapy is initiated, and he is scheduled for formal visual fields. What is the next step? A) Measure GH and IGF-1. B) Neurosurgery consult. C) Radiation therapy consult. D) Cabergoline.

Answer B. Neurosurgery consult. Explanation This is likely a nonfunctioning pituitary tumor with chiasmal compression and lateral extension. He has hypopituitarism and needs adrenal and thyroid replacement. The treatment of choice is surgery. Radiation therapy will not be effective rapidly enough to preserve vision. With a normal PRL, he does not need cabergoline, and clinically, he does not have acromegaly.

A 26-year-old female presents for evaluation of neck pain and fatigue. She describes the pain as located in the front of her neck and does not radiate, but has been constant over the last week. She reports she recently had an upper respiratory infection with cough, nasal congestion, sore throat, and fever last month. Review of systems elicits that the patient has felt fatigued over the last 2-3 weeks as well. On physical exam, her HR is 95, and her BP is 145/87. HEENT exam reveals a clear oropharynx and nares. Thyroid exam reveals a smooth, enlarged thyroid gland that is exquisitely tender to palpation. The remainder of the physical exam is unremarkable. Thyroid function testing reveals: TSH: < 0.01 uU/mL (0.5-5.0 uU/mL) Free T 4: 2.6 ng/dL (0.9-2.4 ng/dL) Radioactive iodine uptake scan reveals decreased uptake throughout the thyroid gland. Which of the following statements regarding the prognosis of the disease is most accurate? A) The patient is likely to have relapse of symptoms. B) No additional therapy is needed. C) The patient will likely require radioiodine ablation. D) Additional imaging would help to direct management. E) The patient will likely require thyroid hormone replacement in the future.

Answer B. No additional therapy is needed. Explanation This patient has subacute thyroiditis (de Quervain's thyroiditis). It is a self-limited disease process that typically occurs after an antecedent trigger like a viral infection and often produces a tender, enlarged thyroid. The classic progression of subacute thyroiditis is a transient hyperthyroid phase followed by a euthyroid phase, hypothyroid phase, ending with a return to a euthyroid state once the underlying inflammation resolves. Only about 15% of patients eventually develop a longer lasting hypothyroid state requiring thyroid hormone replacement. Additional imaging with ultrasound or CT scan is neither necessary nor indicated to manage subacute thyroiditis. Furthermore, the illness is self-limited, and the risk for future episodes is low at about 1.5-4%. Pain treatment with NSAIDs or corticosteroids has been reported, but is often not required as the symptoms will resolve spontaneously.

A new elderly patient comes to your clinic in Philadelphia one winter day complaining about fatigue. His past medical history is insignificant. On examination, you find him to be thin and well tanned, but with sparse pubic hair and no axillary hair. BP 96/62 and HR 109. Chem-7 shows the following: Na+ 138 mEq/L (135-143 mEq/L) K+ 5.3 mEq/L (3.5-5.0 mEq/L) Cl- 108 mEq/L (100-109 mEq/L) HCO3- 19 mEq/L (22-30 mEq/L) Urea nitrogen 9 mg/dL (8-18 mg/dL) Creatinine 0.9 mg/dL (0.6-1.2 mg/dL) Glucose 66 mg/dL (65-110 mg/dL) At this point, you make an initial diagnosis and plan the appropriate workup. Which of the following is the initial diagnosis? A) Pseudo adrenal insufficiency B) Primary adrenal insufficiency C) Secondary adrenal insufficiency D) Hypothyroidism E) Glucocorticoid abuse

Answer B. Primary adrenal insufficiency Explanation This case demonstrates several clues that should lead you toward primary adrenal insufficiency. He has a thin body habitus, decreased body hair, hyperkalemic metabolic acidosis, and a low glucose. This constellation of findings should always make you suspect adrenal insufficiency. The appropriate action is to start dexamethasone while you wait for the results of an ACTH1-24 (cosyntropin) stimulation test. A normal person will have a stimulated cortisol level exceeding 20 mg/dL. His cortisol comes back low, which confirms your diagnosis. Now you must decide between primary and secondary disease. The answer is in the question. How many elderly people in Philadelphia have a good tan in the middle of winter? Yes, he could frequently visit the neighborhood tanning salon, and he could have returned recently from an extended vacation in the Caribbean. But most likely, the well-tanned skin represents hyperpigmentation resulting from elevated levels of melanocyte-stimulating hormone (MSH). ACTH and MSH are derived from POMC. His tan most likely represents the hyperpigmentation associated with excess ACTH production in an attempt to overcome the primary adrenal insufficiency. It would be appropriate to switch him from dexamethasone to hydrocortisone (cortisol) because the latter has mineralocorticoid actions. Remember that a person with primary adrenal insufficiency also loses his mineralocorticoid (aldosterone) production. There is no such thing as pseudo adrenal insufficiency; his clinical presentation is opposite to what you would expect from someone abusing glucocorticoids; and although hypothyroidism could explain his fatigue, it doesn't explain the rest of his presentation.

Your patient is clinically hyperthyroid and has pain in the anterior neck and a smooth, very tender, slightly enlarged thyroid. Free T4: 2.2 (0.9-1.8) TSH: 0.25 (0.5-5.0) Uptake: 1% Thyroglobulin level is low. Which of the following is the most likely diagnosis? A) Exogenous T4 B) Subacute thyroiditis C) Toxic multinodular goiter D) Graves disease

Answer B. Subacute thyroiditis Explanation This combination of clinical signs of hypothyroidism with elevated T4 but reduced RAIU excludes Graves disease but can be seen with subacute thyroiditis, toxic multinodular gland, and excess exogenous T4. A smooth gland on exam does not fit with a toxic multinodular goiter. Exogenous T4 usage does not cause enlargement of the thyroid gland. Nor does it cause painfullness or tenderness of the thyroid. Subacute (painful) thyroiditis does fit all these findings. Subacute thyroiditis is thought to be a post-viral syndrome during which the function of the thyroid goes through several phases: hyperthyroid to euthyroid to hypothyroid then back to euthyroid permantly. Total duration of symptoms is usually 3 - 6 months. Thyroglobulin is usually produced by the thyroid epithelial cells in the production of T3 and T4. With the onset of subacute thyroiditis the thyroglobulin is quickly used up in the making of T4 and T3 and the levels are low (as in this scenario). Thyroglobulin may be elevated in Graves disease, subacute thyroiditis, and with certain thyroid cancers (usually well differentiated papillary and follicular cancer). With cancer, if the thyroglobulin level returns to normal after treatment (half-life = 65 days), it is used as a marker for cancer recurrence.

A patient develops a kidney stone and comes to see you. He is a "health freak" and takes megadoses of vitamins and supplements of calcium. The following labs are all elevated: calcium, ionized calcium, 25-OH-vitamin D, 1,25-(OH)2-vitamin D, and intact PTH. You briefly entertain a long differential diagnosis because of his megadoses of vitamins and minerals, but you quickly discard all of them except one. Which of the following is the most likely diagnosis? A) Vitamin D intoxication B) Primary hyperparathyroidism C) Vitamin A intoxication D) Secondary hyperparathyroidism E) Pseudopseudohypoparathyroidism

Answer B. Primary hyperparathyroidism Explanation The PTH is elevated along with the calcium. Normally, an elevated calcium would suppress the PTH level. His PTH is elevated in the face of hypercalcemia and thus represents primary hyperparathyroidism. The only other rare differential diagnosis is FHH (familial hypocalciuric hyercalcemia), which could be excluded by checking a 24-hour urine for calcium and asking about familial history of hypercalcemia. His vitamin D levels are elevated, but this is due to both his primary hyperparathyroidism and his use of vitamin supplements. Vitamin A intoxication can cause hypercalcemia, but no vitamin A levels are given in this question, and his primary hyperparathyroidism is a better explanation. Pseudopseudohypoparathyroidism refers to a specific phenotype (shortened 4th and 5th metacarpals) with normal calcium and parathyroid hormone levels. His labs rule out such a diagnosis. Secondary hyperparathyroidism refers to the normally elevated levels of parathyroid hormone in the face of hypocalcemia. Low calcium levels will stimulate the parathyroid glands to secrete extra hormone, which will stimulate the conversion of 25-OH-vitamin D into 1,25-(OH)2-vitamin D. His calcium is elevated, so he cannot have secondary hyperparathyroidism.

A 49-year-old man presents to your clinic with a 4-month history of cold intolerance, easy fatigue, depression, some weight gain, and constipation. FT4: 0.5 (0.8-1.8) TSH: 0.5 (0.5-4.0) Which of the following is the most likely diagnosis? A) Primary hypothyroidism B) Secondary hypothyroidism C) Euthyroid sick syndrome D) No thyroid disease

Answer B. Secondary hypothyroidism Explanation This man is presenting with clinical symptoms of hypothyroidism, yet his TSH is not appropriately elevated to compensate for an inadequate amount of T4 (excludes primary hypothyroidism). This occurs in secondary hypothyroidism where disease is in the pituitary gland, and TSH is not released properly. Tertiary hypothyroidism indicates disease in the hypothalamus and a defective production of TSH-releasing hormone. This is not euthyroid sick syndrome because the patient clearly has symptoms of hypothyroidism. Hashitoxicosis specifically refers to the situation of excess production of thyroid hormone due to an immune-mediated mechanism. Hashitoxicosis patients are hyperthyroid, not hypothyroid.

You are asked to see a patient admitted to the medicine service. He was found to have a large pituitary tumor that is extending toward the optic chiasm, but without any significant changes in his visual fields. The tumor is a prolactinoma. The primary physician is very concerned because of the size of the tumor. He wants to have the patient operated on immediately. What would be the next best thing to do? A) Transfer the patient to a hospital with a competent neurosurgeon. B) Start cabergoline and repeat the CT/MRI in a few days. C) Begin a trial of high-dose glucocorticoids. D) Begin pre-op evaluation in preparation for surgery in 3 days.

Answer B. Start cabergoline and repeat the CT/MRI in a few days. Explanation This patient has a large prolactinoma, and the primary physician wants it to be removed. However, in the absence of visual field defects, seizures, severe headaches, and other findings, the surgery is not emergent and can probably wait a few days. In the meantime, start the patient on cabergoline. This drug may shrink the tumor enough to avoid surgery. Though it usually takes much more than a few days to show an appreciable reduction in size, the prolactinoma may nevertheless begin to shrink within a short period of time. Beginning the preoperative evaluation is justified, but it is more important to begin cabergoline. High-dose glucocorticoids might be justified if the patient presented with neurologic symptoms, but they are not needed in this case.

A 66-year-old woman recently diagnosed by her cardiologist 3 months ago with Type 2 diabetes mellitus is in your office for a routine follow-up. She also has stage III congestive heart failure, which is closely followed by her cardiologist, who recently placed a note in her chart stating that the CHF is currently stable. She follows an American Heart Association step 3 diet and a cardiologist-approved walking program as well as she can. She takes her prescription drugs exactly as instructed, including metformin 500 mg at bedtime. She steadfastly refuses to take insulin. Her glycemic control is less than desired. The fasting glucose levels average 90-100, postprandial levels 200-210, and bedtime levels 100-120. The HgbA1c is 8.2 %, serum creatinine is 1.5 mg/dL, AST/ALT are normal, and bilirubin is normal. Which of the following should be used to improve her glycemic control? A) Add pioglitazone 45 mg once daily or rosiglitazone 4 mg twice daily. B) Stop metformin and begin repaglinide 1 mg before meals. C) Add acarbose 50 mg with every meal. D) Increase metformin to 1,000 mg twice daily in steps as tolerated. E) Stop metformin and begin either pioglitazone or rosiglitazone.

Answer B. Stop metformin and begin repaglinide 1 mg before meals. Explanation Given current guidelines, the correct answer for board exams from the list of possible answers is to stop metformin and begin repaglinide. Every answer that continues metformin is incorrect because she has a contraindication to the drug. Metformin's rate of elimination is decreased in renal insufficiency. Guidelines say not to be used in any man with a stable creatinine ≥ 1.5 mg/dL or any woman with a creatinine ≥ 1.4 mg/dL. The woman in this scenario has a creatinine of 1.5 mg/dL. In addition, the thiazolidinediones—pioglitazone and rosiglitazone—are not approved for patients with stage III or IV CHF. She would be a good candidate for fast-acting insulins like insulin aspart or lispro with meals, but she refuses insulin. But do know that many experts base the metformin decision on creatinine clearance (CCl) formulas that give a better estimation of GFR. Using these formulas, an estimated GFR < 30 mL/min is an absolute contraindication with 30-60 mL/min being an area during which patients are carefully watched and metformin dosage is decreased. CCl = 46.8 per the Cockcroft-Gault equation [(140-age) x lean body wt (kg)/(cr x 72)] (lean body weight in women is 0.85 x wt). For board exam questions, answer per the renal tests that are done (Cr vs GFR/CrCl).

You are covering for a colleague one week when a diabetic comes to see you for her routine visit. She is 53 years old, has a history of diabetic neuropathy, and has an early foot ulcer. She checks her glucose and reports that her morning glucose is generally 90-120. She takes metformin 500 mg twice daily and a baby aspirin every day. She also has been taking cephalexin, prescribed by an infectious disease expert who is following the foot ulcer closely. She doesn't smoke and her blood pressure is 130/78. Her recent lab results are as follows and are not much different from her last visit: Na+ 140 mEq/L (135-143 mEq/L) K+ 4.2 mEq/L (3.5-5.0 mEq/L) Cl- 104 mEq/L (100-109 mEq/L) HCO3- 24 mEq/L (22-30 mEq/L) Urea nitrogen 15 mg/dL (8-18 mg/dL) Creatinine 1.4 mg/dL (0.6-1.2 mg/dL) Glucose 143 mg/dL (65-110 mg/dL) Total bilirubin 0.3 mg/dL (0.1-1.0 mg/dL) HbA1c 8.1 % (4.9-6.2 %) LDL 140 mg/dL HDL 34 mg/dL TG 179 mg/dL WBC 9.8 × 103/µL (4.5-11.0 × 103/µL) Which of the following is the most important action for you to take at this time? A) Add an additional oral agent. B) Stop the metformin and start a different oral agent. C) Begin a statin. D) Increase the metformin to 1,000 mg twice daily. E) Add an antibiotic that offers Pseudomonas coverage.

Answer B. Stop the metformin and start a different oral agent. Explanation Contraindications for metformin therapy: It is contraindicated in a man with a creatinine ³ 1.5 mg/dL or in a woman with a creatinine of 1.4 mg/dL. Start treating her with something else. Adding another agent or increasing the metformin dose will improve her glycemic control, but these other choices do not discontinue her metformin. An ID specialist is watching her foot ulcer closely so there is no reason to change the antibiotic. Adding a statin is a good idea, but it is not as important as discontinuing her metformin at this time.

During the routine evaluation of a 37-year-old female, you note a palpable lesion in the right lobe of the thyroid. The patient has been asymptomatic and was unaware of the lesion. It is firm and mobile, with distinct borders. No other lesions or abnormalities are detected. Which of the following would be the most appropriate next step in the evaluation of this nodule? A) Thyroid ultrasound B) TSH C) Thyroid scan D) Fine needle aspiration

Answer B. TSH Explanation A thyroid nodule has the potential to be a malignant or benign process, and differentiation is critical. The first step in the evaluation is measurement of TSH levels. This will help determine which direction to proceed: If the TSH is low, indicating hyperthyroidism, then proceed with a scan. If the TSH is normal or high, most proceed with ultrasound and fine needle aspiration if clinically suspicious. Enlargements associated with hyperthyroidism are unlikely to be malignant in nature. Fine needle aspiration is performed for lesions that are associated with normal thyroxine levels or those that are found to be "cold" nodules.

You are asked to see a woman recently hospitalized for a hip fracture and found to have hypocalcemia. She is 75 years old and has been in poor health for quite some time. She lives alone and is reluctant to leave her house. Her granddaughter does the shopping and laundry for her, but admits that her grandmother doesn't eat very much. Her only medication is hydrochlorothiazide 50 mg every morning for mild hypertension; she rarely goes to her internist for scheduled appointments. When you examine her, you find a frail-looking elderly woman with moderate dementia. She has a right hip fracture that she attributes to falling down the steps to her front door. The orthopedic surgeon is currently deciding whether to operate. In the meantime, full DVT precautions are in place. Her labs, including the additional tests ordered by you, come back as the following: Na+ 142 mEq/L (135-143 mEq/L) K+ 3.6 mEq/L (3.5-5.0 mEq/L) Cl- 105 mEq/L (100-109 mEq/L) HCO3- 28 mEq/L (22-30 mEq/L) Urea nitrogen 22 mg/dL (8-18 mg/dL) Creatinine 1.2 mg/dL (0.6-1.2 mg/dL) Glucose 86 mg/dL (65-110 mg/dL) Calcium 7.6 mg/dL (8.9-10.5 mg/dL) Phosphate 1.8 mg/dL (2.5-4.5 mg/dL) Magnesium 2.9 mg/dL (1.4-2.5 mg/dL) 25-OH-vitamin D 6 μg/L (10-55 μg/L) 1,25-(OH)2-vitamin D 12 ng/L (18-62 ng/L) Intact PTH 81 pg/mL (10-65 pg/mL) Bone density at hip z-score is -2.8 SD. Which of the following is the most likely cause of her hypocalcemia? A) Hypermagnesemia B) Vitamin D deficiency C) Primary hyperparathyroidism D) Thiazide diuretic E) Hypophosphatemia

Answer B. Vitamin D deficiency Explanation This lady has probably had hypocalcemia for quite some time and came to medical attention only because she experienced an osteoporotic hip fracture. An extremely common cause of hypocalcemia is vitamin D deficiency, and she exhibits a classic presentation. Her 25-OH-vitamin D level is low, reflecting both her poor diet and her low exposure to sunlight due to her reluctance to leave the house. Hypocalcemia normally gives rise to an elevated parathyroid hormone level (secondary hyperparathyroidism) and this in turn stimulates the conversion of 25-OH-vitamin D to 1,25-(OH)2-vitamin D. Unfortunately, her levels of 25-OH-vitamin D are already low, and her conversion to 1,25-(OH)2-vitamin D is thus impaired. The phosphate level is low because of the hyperparathyroidism, which stimulates renal excretion of phosphates. While hypomagnesemia can cause hypocalcemia due to impaired secretion of parathyroid hormone, her hypermagnesemia is not contributing. Hyperphosphatemia may cause hypocalcemia because of local precipitation of calcium-phosphate salts. Her hypophosphatemia is not the cause of the hypocalcemia. Thiazide diuretics are associated with hypercalcemia, not hypocalcemia. Primary hyperparathyroidism causes hypercalcemia, not hypocalcemia.

A 70-year-old man with steroid-dependent asthma is admitted with fever, congestive heart failure, atrial fibrillation, and episodes of confusion. On exam, his thyroid is smooth and three times enlarged. His free T 4 is 6.3 (0.9-1.5), and his TSH is suppressed. In the ICU, he has received intravenous beta-blocker therapy and 100 mg of hydrocortisone. What is the most important next step in treating the hyperthyroidism? A) Arrange for a technetium scan. B) Administer cold iodine. C) Administer methimazole. D) Administer radioactive iodine.

Answer C. Administer methimazole. Explanation This is a description of thyroid storm. The first step is to block thyroid hormone synthesis. The second step is to block release of thyroid hormone from the gland. An antithyroid agent is given first, followed 1 hour later by cold (nonradioactive) iodine such as Lugol's solution. Treatment also involves decreasing the conversion of T 4 to T 3 and glucocorticoids do that. Radioactive iodine may be definitive treatment after the thyroid storm resolves but is not appropriate in the emergent setting. PTU decreases the conversion of T 4 to T 3 but is not recommended because of hepatotoxicity.

A 30-year-old male has gained 30 pounds, developed muscle weakness, hypertension, and bruising. After an overnight dexamethasone suppression test, his 8 a.m. cortisol was 15. Last week an 8 a.m. ACTH was 1 (10-60). Urine free cortisol levels measured before and after 2 and 8 mg of dexamethasone were: baseline: 1,400 µg 2 mg: 1,300 µg 8 mg: 1,200 µg What is the most likely etiology of the glucocorticoid excess in this patient? A) Pituitary adenoma B) Ectopic ACTH syndrome C) Adrenal adenoma D) Hypothyroidism

Answer C. Adrenal adenoma Explanation The failure to suppress cortisol after dexamethasone could be due to ectopic ACTH production or to an adrenal adenoma, but only an adrenal adenoma would have a suppressed ACTH. Hypothyroidism would not have this effect on cortisol levels.

A 37-year-old man is referred to you for cholesterol of 256 mg/dL. He states he has decreased intake of meat, fried foods, and dairy other than low- or no-fat products, and has trouble jogging due to leg cramps. Despite diet change, his weight increased 5 pounds in the past 3 months. Pulse is 66 and BP is 146/88. His thyroid is difficult to feel in his large neck. His hair is not shiny, his skin is dry, and there are no bruits, xanthomas, or xanthelasma. He returns in 2 weeks, training for a half triathlon, with his LDL 196 and triglycerides 123; total cholesterol is 252 and HDL 21. Which of the following pharmacologic treatments is best to begin at this time? A) Nothing; wait longer for diet to take effect B) Nitroglycerin C) Atorvastatin D) Bile acid resins E) Gemfibrozil

Answer C. Atorvastatin Explanation The point here is that he needs LDL treated and, if possible, HDL raised. He is already exercising, which is an excellent means to raise HDL.

A 60-year-old woman is establishing care in your clinic. She has occasional numbness and tingling in her fingers and toes and has noted some numbness around her mouth, especially when she is stressed or anxious. She had thyroid surgery for Graves disease about 2 years ago and takes 100 µg of levothyroxine and 1 tablet of calcium daily. On exam, her blood pressure is 130/80, pulse 80, and she has cramping in her right forearm and fingers when the blood pressure cuff is attached. Based on this history and exam, which of the following is most likely? A) Calcium 8.5, PTH 65, PO4 4.5 B) Calcium 9.5, PTH 35, PO4 4.0 C) Calcium 6.0 (8.5-10.5), PTH 2 (10-65), PO4 6.0 (2.7-4.5) D) Calcium 10.8, PTH 108, PO4 2.3 E) Calcium 8.0, PTH 98, PO4 2.1

Answer C. Calcium 6.0 (8.5-10.5), PTH 2 (10-65), PO4 6.0 (2.7-4.5) Explanation This is a classic presentation of surgically induced hypoparathyroidism. The PO4 is elevated because of the absence of PTH, and the PTH is virtually undetectable. The other options are incorrect either because the calcium is not low, the PTH is elevated, or the PO4 is normal.

A 30-year-old woman complains of increased thirst and nocturia. She has a normal exam, and her electrolytes, calcium, and glucose are normal. Following an overnight fast, her urine specific gravity was 1.020, and her urine osmolality was 700. What should you do now? A) Begin DDAVP®. B) Begin amiloride. C) Reassurance. D) H 2 O deprivation test.

Answer C. Reassurance. Explanation Increased thirst and polyuria are common complaints, but diabetes insipidus is rare. The first step is to determine if the patient can concentrate her urine. A healthy person (with normal ADH) will increase urine specific gravity and urine osmolality when H 2 O is withheld. Her urine specific gravity and urine osmolality responded appropriately to the fast, so she needs no further testing or treatment.

A 47-year-old febrile, diabetic man presents to the emergency department after being found down by his wife. Initial evaluation reveals a left lower lobe pneumonia. Admission labs: Na 128, K 3.2, Cl 98, HCO3 7, Glu 700 He is given 2 L of 0.9% NaCl, a bolus of insulin, and started on a continuous insulin infusion. Repeat labs: Na 136, K 3.0, Cl 105, HCO3 19, Glu 375 Which of the following is the most appropriate next step with regard to insulin and IVFs? A) Change to 5% dextrose with 0.45% NaCl and continue the insulin infusion at the current rate. B) Continue the current IVF and double the insulin infusion rate. C) Continue the current management. D) Give 10 U regular insulin, stop the IVF and the insulin infusion, and allow the patient to eat. E) Give 10 U regular insulin, stop the IVF, continue the insulin infusion for 2 hours at the current rate, then stop.

Answer C. Continue the current management. Explanation The initial laboratories reveal hyperosmolar hyponatremia with an elevated anion gap. The sodium corrects into normal range when considering the hyperglycemia. To correct sodium for hyperglycemia, increase the sodium by 1.6 for each 100 increment in glucose over 100: Measured glucose - Normal glucose = 700 - 100 = 600, or 6 increments over 100. 6 x 1.6 = 9.6. To correct the sodium, add 9.6 to the measured sodium (128) = 137.6 = Normal. Anion gap = 128 - 98 - 7 = 23. (Although controversial, consensus is to use the measured serum sodium concentration when calculating the anion gap.) The repeat labs show correction of the anion gap, but the patient remains hyperglycemic. New anion gap = 136 - 105 - 19 = 12. Therefore, the acidosis has resolved, but hyperglycemia is still present. The question next queries if you know when to change the IVF to a dextrose-containing fluid. The patient's blood glucose is 375, and recommendations state that the fluids should be supplemented with dextrose when the serum glucose concentration is < 200 mg/dL. Therefore, none of the answers that suggest changing the fluids or stopping the fluids is correct. Because the patient's high anion gap has resolved and the glucose has lowered by > 70 mg/dL, the rate of insulin infusion is adequate, and increasing the insulin rate is unnecessary. Current management should be continued until the blood glucose is < 200 mg/dL, at which time dextrose-containing fluids should be substituted, and the insulin infusion should be titrated to a blood glucose of 100-200 mg/dL. Once the glucose is 100-200 mg/dL, and the anion gap remains in the normal range, the IV insulin is continued until the patient is able to eat. At that time, give a subcutaneous injection of regular insulin, continue the infusion for 1-2 hours after the injection, and allow the patient to eat. Titrate the regular insulin to the blood sugar and initiate long-acting insulin when the insulin infusion is discontinued.

An elderly woman comes to your clinic complaining of weakness, fatigue, and having no interest in life. On questioning, she reports cold intolerance, 1 bowel movement every 4-5 days, and some hair loss, but denies any weight gain. She is depressed and doesn't care about herself or her home. BP 98/62 and HR 57. Laboratory: Na+ 140 mEq/L (135-143 mEq/L) K+ 4.0 mEq/L (3.5-5.0 mEq/L) Cl- 109 mEq/L (100-109 mEq/L) HCO3- 24 mEq/L (22-30 mEq/L) Urea nitrogen 9 mg/dL (8-18 mg/dL) Creatinine 0.9 mg/dL (0.6-1.2 mg/dL) Glucose 72 mg/dL (65-110 mg/dL) FT4 0.3 ng/dL (0.7-1.5 ng/dL) TSH 2.36 mU/L (0.5-5.0 mU/L) Which of the following should you do now? A) Begin IV levothyroxine. B) Begin oral levothyroxine. C) Do a cosyntropin stimulation test. D) Begin hydrocortisone.

Answer C. Do a cosyntropin stimulation test. Explanation This is a case of secondary hypothyroidism. The teaching point is to recognize that she likely also has secondary adrenal insufficiency and it is important to make the diagnosis and treat adrenal insufficiency before starting thyroid hormone. Starting IV levothyroxine is wrong because you would not give IV T4 to an elderly individual, especially one who might have adrenal insufficiency. Beginning hydrocortisone is wrong because you would not give hydrocortisone until it has been confirmed that the patient actually has adrenal insufficiency. This question requires you to understand Schmidt syndrome. And this question tests your ability to consider the entire presentation and not to focus on the obvious. Of course she is clinically and biochemically hypothyroid, but don't miss the other possible issue. She has several clues that raise the possibility of adrenal insufficiency: thin body habitus, not gaining weight despite being hypothyroid, low blood pressure, and a hint of hyperkalemic metabolic acidosis. The blood pressure is not unusual for a woman, but hypothyroidism usually raises diastolic blood pressure. The blood pressure alone is not diagnostic of adrenal insufficiency, but it is highly suggestive when taken in context with her other findings. The glucose is also normal, but how many of your patients have a fasting glucose this normal? The board examination is not likely to give you an obvious case of adrenal insufficiency, so be suspicious whenever you encounter a high-normal potassium with a somewhat low bicarbonate. It is imperative to recognize a possible case of Schmidt syndrome (combination of hypothyroidism and adrenal insufficiency) because the patients die soon after starting thyroid hormone replacement unless they begin glucocorticoid replacement. If you ever have any suspicion that a patient may have adrenal insufficiency, you must begin steroids and work up the patient. The standard test is an ACTH1-24 (cosyntropin) stimulation test. Because dexamethasone is the only available glucocorticoid that does not interfere with the cortisol assay, the patient must be started on dexamethasone until the results of the test are available. If she truly has adrenal insufficiency, it should be treated with hydrocortisone because it also has some mineralocorticoid activity. Once she takes her first steroid pill, she can begin thyroxine.

A 21-year-old man is brought into the emergency department after being run over by a motorcycle. On routine lab he is found to have a serum calcium of 11.5 mg/dL and a phosphorus of 3.0 mg/dL (normal is 2.5-4.5 mg/dL). He remembers that his mother told him that his sister or brother might have a "high calcium," but that is all he knows. His examination is normal except for a tire mark on his chest. LABORATORY: Serum intact PTH of 72 pg/mL (normal 10-65 pg/mL) 1,25-dihydroxyvitamin D of 40 pg/mL (normal 15-60 pg/mL) 24-hour urinary calcium of 30 mg (normal 100-300 mg/24 hour) What is the appropriate therapy at this time? A) Low calcium diet. B) Steroids. C) Do nothing at this point. D) Bilateral neck exploratory surgery. E) Oral phosphates.

Answer C. Do nothing at this point. Explanation This guy has familial hypocalciuric hypercalcemia (FHH), also known as benign familial hypercalcemia. This consists of an elevated serum calcium, normal or low serum phosphorus, a normal or high serum PTH, a slightly elevated magnesium, and a low urinary calcium excretion. This is inherited as an autosomal dominant disorder of chromosomes 3 or 19. No therapy is required. Surgery will not correct the hypercalcemia. The other thing to be careful about is that these patients can be misdiagnosed as having hyperparathyroidism if the urinary calcium is not measured. Untreated, these patients generally remain asymptomatic without evidence of renal calculi, HTN, or other problems with hypercalcemia.

A 55-year-old woman comes to your clinic for a routine visit. She has rheumatoid arthritis. Her rheumatologist has been treating her for the last 2 months with 5 mg of prednisone daily and intends to continue this therapy for the foreseeable future. She has a history of esophageal stricture. She gets limited exercise and avoids sun exposure as much as possible because it worsens her skin manifestations. To lower her risk of developing glucocorticoid-induced osteoporosis, which of the following do you recommend? A) Begin calcium 4 g per day. B) Begin an oral bisphosphonate. C) Encourage weight-bearing exercises. D) Continue to discourage sun exposure because of risk of melanoma. E) Begin vitamin D 100 units per day.

Answer C. Encourage weight-bearing exercises. Explanation This question requires you to know the recommendations for preventing glucocorticoid-induced osteoporosis. Glucocorticoids directly weaken bone by stimulating bone resorption and inhibiting bone formation. They have a negative impact on calcium by decreasing gastrointestinal absorption and increasing renal excretion. This in turn may increase PTH, which will further weaken bones. Nevertheless, calcium and PTH levels generally remain within normal limits, suggesting that the main problems are the direct effects of glucocorticoids on bone. The following are recommended by the American College of Rheumatology for preventing glucocorticoid-induced osteoporosis in patients starting prednisone equivalents at least 5 mg daily for 3 months or less: Modify lifestyle Weight-bearing exercises Calcium 1.5 g daily Vitamin D 800 units daily Oral bisphosphonates (caution in premenopausal women) The following are recommended by the American College of Rheumatology for treating glucocorticoid-induced osteoporosis in patients receiving prednisone equivalents at least 5 mg daily for at least 6 months: Modify lifestyle Weight-bearing exercises Calcium 1.5 g daily Vitamin D 800 units daily Monitor BMD and treat T-scores of -1 or less Monitor for hypogonadism and consider replacement therapy if needed Bisphosphonates if T-score is -1 or less or fracture occurs (caution in premenopausal women) Treat with calcitonin if other drugs are contraindicated. The patient in this question should not be told to consume 4 g of calcium daily, and vitamin D of 100 units daily is too small of a dose. She cannot take bisphosphonates because they are contraindicated in someone with a history of esophageal stricture. She should get some sun exposure as well.

A 32-year-old woman presented to the emergency department with pain in the RUQ, fever, and jaundice. She was diagnosed with ascending cholangitis and admitted to the ICU. After stabilizing, her heart rate was recorded as 55-65 bpm for the past 2 days. Thyroid studies were ordered with the following results: TSH: 0.20 (0.5-5.0), Free T 4 : 1.7 (5-12) You are consulted for recommendations on the need for further evaluation and institution of thyroid replacement hormone. Which of the following is the most appropriate next step in patient care? A) Start levothyroxine 100 micrograms daily. B) Diagnose secondary hypothyroidism and order an MRI of the pituitary. C) Reassure the consulting physician. D) Order a thyroid uptake scan. E) Start levothyroxine 50 micrograms daily.

Answer C. Reassure the consulting physician. Explanation Remember that you don't touch people's thyroids if they are sick or just had surgery, unless the initial presentation suggests myxedema coma or thyroid storm. This patient presented with gallstones. Any thyroid abnormalities postoperatively should be considered euthyroid sick syndrome, and labs should be repeated at a later date to confirm reversion to normal functioning.

A 35-year-old transcriptionist is referred to you for evaluation. During the past 4 months, she has noted increasing fatigue, weight loss of 12 pounds despite a good appetite, and increased diaphoresis. Recently, she has noticed a tremor in her right hand that makes typing more difficult. Also, she has had severe mood swings and frequently gets angry with herself for outbursts that she makes to other people. FAMILY HISTORY: Mother with surgery for "overactive" thyroid PHYSICAL EXAMINATION: Firm, nontender thyroid enlargement about twice normal size Fine tremor of outstretched arms Generalized hyperreflexia LABORATORY: Serum Thyroxine: 15.0 micrograms/dL (high) Serum T3: 290 ng/dL (high) Resin T3 uptake: 40.1% (high) TSH: < 0.1 μIU/mL Radioactive iodine uptake (RAIU): 24-hour uptake of 2.7% (normal 5-25%) All of the following are possible diagnoses for this patient except: A) Struma ovarii B) Excess iodine ingestion C) Graves disease D) Silent thyroiditis E) Subacute thyroiditis

Answer C. Graves disease Explanation In this case, realize that Graves disease is unlikely! The thyrotoxic phase of subacute and silent thyroiditis is associated with an inflammatory breakdown and release of stored thyroid hormone, which will feed back onto the hypothalamus and anterior pituitary to inhibit TSH secretion. This will produce a low RAIU for a while. Struma ovarii, remember, consists of ectopic hyperfunctioning autonomous thyroid tissue—like in a teratoma, which also will result in a low RAIU in the thyroid (however, if the pelvis is scanned, it would show an increased uptake here). Iodine-induced thyrotoxicosis will generally occur in the face of a preexisting dysfunctional thyroid. We see this mainly in someone who is coming from a country with low iodine intake, to the USA with our high iodine intakes.

A 40-year-old man presents with several unique complaints. He has noted for some time that his voice was deepening and that he seems to sweat a lot. He went skiing last week, and, when he placed his helmet on his head, he noted that it no longer fit properly—the helmet seemed to be too small. On physical examination you note frontal bossing and coarse facial features. Thinking you know the diagnosis, which test would help you confirm it? A) Simultaneous glucose and growth hormone levels at bedtime and 4 a.m.; look for a 3-fold rise in growth hormone. B) Random growth hormone level; should be > 5 ng/mL in this patient. C) Growth hormone level 1 hour and 2 hours after 100-gram glucose load; this should be > 5 ng/mL in this patient. D) Growth hormone levels after a 12-hour fast; look for growth hormone > 5 ng/mL. E) Growth hormone level 1 hour and 2 hours after 100-gram glucose load; this should be < 5 ng/mL in this patient.

Answer C. Growth hormone level 1 hour and 2 hours after 100-gram glucose load; this should be > 5 ng/mL in this patient. Explanation He most likely has acromegaly. Deepening voice, excessive sweating, frontal bossing, coarse facial features and enlarging hands/feet are the classic findings. Diagnosis is made by either an oral glucose tolerance test or by measuring IGF-1 levels. A random growth hormone level is likely not diagnostic given the pulsatile nature of growth hormone secretion. A low glucose will normally cause an elevated growth hormone level. High glucose levels during an oral glucose tolerance test should normally suppress growth hormone—so, high growth hormone levels after a glucose load are consistent with acromegaly.

A 70-year-old woman presents with complaints of constipation and leg and back pain. She had problems with rectal bleeding 6 months ago and was found to have a polyp on colonoscopy. This was removed, and the site cauterized. At the time of workup, she had a Hct of 27% with an MCV of 78. She was started on FeSO4. Two months ago, she returned for her annual exam and was found to be in good shape except for complaints of constipation and pruritus. Her cholesterol was 300, and she was started on simvastatin. Today, she has even more problems with constipation than before. Her pruritus continues. MEDICATIONS: Simvastatin 20 mg qd, FeSO4 325 mg tid, levothyroxine sodium 0.1 mg qd, ranitidine, psyllium 1 TBS qd, felodipine 10 mg qd. On exam, VS: BP 120/70; P 55; Chest clear; Abd soft, tympanitic; Skin—xerosis present; Rectal—heme-negative; Ext—tenderness over the muscles on palpation. Which of the following options would best explain her symptoms? A) Colon cancer B) Drug interaction between simvastatin and felodipine C) Interaction between levothyroxine sodium and FeSO4 D) Interaction between ranitidine and simvastatin E) Rhabdomyolysis due to HMG CoA reductase inhibitor

Answer C. Interaction between levothyroxine sodium and FeSO4 Explanation This patient has a history of hypothyroidism treated with levothyroxine. She develops iron-deficiency anemia and is started on iron. She then develops symptoms and signs of hypothyroidism: dry skin, elevated cholesterol, constipation, and muscle pain. The reason for her hypothyroidism is poor absorption of the levothyroxine she is taking due to an interaction between the oral iron and the levothyroxine.

After an extremely difficult delivery of her second child, a 26-year-old female developed persistent amenorrhea and is now taking estrogen and progesterone. In the last 6 months, she has noted constipation, fatigue, dry skin, and her weight has increased by 10 pounds. Thyroid function studies revealed free T4 0.6 (0.8-1.9) and TSH 0.6 (0.2-4.2). What is the most appropriate next step? A) Measure FSH and estradiol. B) Measure T3. C) Measure cortisol. D) Measure IGF-1 and growth hormone.

Answer C. Measure cortisol. Explanation The clinical picture suggests Sheehan syndrome, which can affect all hormones in the pituitary, but the ones that are most important are TSH and ACTH. Her symptoms sound like hypothyroidism, but measuring T3 is not necessary to confirm the diagnosis of hypothyroidism. Measuring FSH and estradiol will not be helpful, as you already know she has amenorrhea, and GH deficiency is not clinically important in a 26 year old. She needs levothyroxine, but it is first necessary to screen for secondary adrenal insufficiency and the easiest way to do that is to check a cortisol.

A 26-year-old woman comes to your clinic complaining of a neck mass. The mass does not hurt, but it has been present for 3 months without going away. In fact, she thinks it is now a little larger. She denies dysphagia, odynophagia, breathing difficulty, or changes in her voice. There is no history of exposure to radiation. She is afebrile and clinically euthyroid. The mass is a 1.8-cm nodule in the lower right pole of her thyroid gland. You order thyroid tests and have her return in one week for a fine needle biopsy. The TSH and FT4 are within normal limits. She returns in one week, and you inform her that she has medullary thyroid cancer. You schedule a thyroidectomy. On further questioning, you learn that her mother died at age 39 from an unknown cause. She has two sisters, one brother, and a 2-year-old son. Which laboratory test should be ordered to help assess the risk of her first-degree relatives developing medullary thyroid cancer? A) Serum gastrin B) Serum calcium C) Ret-proto-oncogene (RET) D) Serum calcium after administering gastrin E) Serum calcium after administering calcitonin

Answer C. Ret-proto-oncogene (RET) Explanation This question requires you to know the usefulness of the ret-proto-oncogene in cases of medullary thyroid cancer. Medullary thyroid cancer may be sporadic or familial. The biggest issue with familial disease is identifying family members who may be at risk. The preferred test is the ret-proto-oncogene. Hundreds of mutations have been identified. Though some mutations seem to be associated only with sporadic disease and some with familial disease, there is a tremendous degree of overlap—and few mutations can virtually eliminate familial disease. However, if the patient has an identifiable mutation, all first-degree relatives should be tested. Any family member with the same mutation either already has or will develop medullary cancer. These patients require a thyroidectomy regardless of age. Even her 2-year-old son would need surgery if he has the mutation. But if the patient has a mutation and a family member does not, then that family member has no risk and need not be tested again. A problem develops when no ret-proto-oncogene mutation can be documented in the index case. An older test utilized for screening family members was the pentagastrin-stimulated calcitonin level. This test detects only existing medullary thyroid cancer, and all first-degree relatives had to be tested periodically for much of their lives. (This test is not currently available because pentagastrin is no longer being manufactured.) The other tests listed in the question do not help in identifying family members at risk

A 22-year-old woman presents with weight loss and fatigue. She had an episode of near syncope which brought her to seek medical attention. Her last menstrual cycle was 4 months ago. She complains of abdominal pain, nausea, and anorexia. Her older sister has Type 1 diabetes and primary hypothyroidism. Supine BP is 100/60, HR 92, but standing BP is 66/40. Abdomen is soft and nontender. Urine β-HCG is negative. Laboratory studies: Urinalysis is unremarkable Serum sodium is 128 mEq/L Which of the following is most likely to lead to the correct diagnosis? A) Measure serum potassium. B) Measure free T4 and TSH. C) Measure serum cortisol before and after cosyntropin (Cortrosyn™) injection. D) Measure plasma glucose. E) Measure HbA1c.

Answer C. Measure serum cortisol before and after cosyntropin (Cortrosyn™) injection. Explanation Key points: Hypotension, weight loss, chronic abdominal pain, and hyponatremia are the classic presentation for Addison disease or primary adrenal insufficiency. The family history of other autoimmune endocrine failures is an important clue. The absence of polyuria/polydipsia and the absence of glycosuria make new-onset Type 1 diabetes with dehydration quite unlikely. Secondary amenorrhea is a nonspecific consequence of cortisol deficiency. Since her urine is already known not to contain glucose, it is unlikely that she will have the degree of hyperglycemia needed to cause severe dehydration and hypotension or to elevate HbA1c. ~ 60% of patients with Addison's will have hyperkalemia, but measuring the serum potassium will not directly lead to the correct diagnosis and could be misleading if normal. Hyper- or hypothyroidism would not cause hypotension.

A 27-year-old woman with Type 1 diabetes is taking 15 units of NPH and 10 units of regular insulin before breakfast and around 6 p.m. She cannot afford a more expensive insulin preparation. Her exam is normal, and the table below is a record of her blood sugars recorded at home. At least once per week, she awakens around 3 a.m. with hypoglycemia. 3 A.M. FASTING LUNCH SUPPER BEDTIME Day 1: 90 240 140 110 120 Day 2: 100 220 130 120 140 Which of the following adjustments will improve her blood sugar control? A) Increase the p.m. NPH to 20 units. B) Increase the a.m. regular insulin to 15 units. C) Move the p.m. NPH to bedtime. D) Move her p.m. regular to bedtime.

Answer C. Move the p.m. NPH to bedtime. Explanation The NPH peak effect occurs at 8 hours. If she takes it at bedtime, she will avoid early a.m. hypoglycemia, and the fasting sugars should improve.

An 80-year-old man has been hospitalized for 1 month after treatment of a perforated ulcer. He has no history of thyroid disease. Over the last week, he has noted cold intolerance, dry skin, and periorbital puffiness. His surgeons obtained a FT 4 ↓ , TSH↓, and T 3 ↓. You are asked to treat the hypothyroidism. What do you recommend? A) Repeat FT 4 , TSH, and T 3 in one week. B) 10 µg liothyronine. C) No intervention. D) 0.025 mg levothyroxine.

Answer C. No intervention. Explanation This is a clinical picture of thyroid function studies seen in many hospitalized patients. Usually T 3 levels decline, FT 4 may decline or stay normal, and the TSH may be low or normal. Treatment is not indicated because the levels will normalize after the illness has abated. This is sometimes referred to as the euthyroid sick syndrome, and the levels should be repeated 3 months after discharge.

A 24-year-old female has had no menstrual periods for 15 months and has dark hair on her face, chest, lip, breasts, and lower abdomen. Her pelvic exam shows an enlarged clitoris. Testosterone is 220 (6-82); DHEA-S 200 (200-335); and her A1c is 7.8%. Which of the following is the most likely cause of the hirsutism? A) Idiopathic hirsutism B) Congenital adrenal hyperplasia C) Ovarian tumor D) Adrenal tumor E) Polycystic ovarian syndrome

Answer C. Ovarian tumor Explanation The most important observation is that she is virilized. The clitorimegaly and high testosterone can only mean an ovarian tumor. If she had an adrenal tumor, the DHEA-S would be very high. Women with PCOS are rarely virilized. Idiopathic hirsutism is not associated with virilization.

During routine laboratory screening for an asymptomatic healthy adult, the following lipid values were obtained: Cholesterol Total: 222 mg/dL < 200 Desirable 200-239 Borderline High ≥ 240 High LDL: 137 mg/dL < 100 Optimal 100-129 Near or above normal 130-159 Borderline High 160-189 High ≥ 190 Very High HDL: 75 mg/dL < 40 Low ≥ 60 High Triglycerides: 1,820 mg/dL < 150 Based on these laboratory findings, which of the following abnormalities would be the most likely complication? A) Xanthelasmata B) Hypernatremia C) Pancreatitis D) Nephrotic syndrome E) Hepatic cirrhosis

Answer C. Pancreatitis Explanation Hypertriglyceridemia with levels over 1,000 mg/dL are a risk factor for the development of pancreatitis. Triglyceride levels are occasionally elevated secondary to other medical conditions such as nephrotic syndrome, cirrhosis, thyroid deficiency, etc., but this is a response to these processes and not a cause. Elevated triglyceride levels can also interfere with measurement of sodium, causing a spurious hyponatremia, but hypernatremia is not a direct result of increased triglyceride levels. A xanthelasma may develop based on a genetic predisposition or in association with elevated cholesterol levels. Triglyceride levels, however, are not associated with this condition.

A 27-year-old woman presents with complaints of easy fatigability and recent occurrence of milky discharge from her nipples. She is the mother of an 11-month-old girl but has not nursed in 6 months. She has not started to menstruate since the pregnancy. The discharge began about 6 weeks ago. She denies headaches or visual disturbances. She is on no medications currently. Physical examination is normal except for the presence of galactorrhea. Her prolactin level is 208 ng/mL (elevated). Which of the following statements is true? A) Empty-sella syndrome needs to be investigated. B) Hypothyroidism is likely. C) Pregnancy must be ruled out. D) This is a common, benign, post-pregnancy condition. E) Most likely she has a prolactinoma.

Answer C. Pregnancy must be ruled out. Explanation This is a reminder to always think about the "common" things first. She has galactorrhea and her prolactin level is elevated at 208 ng/mL. Prolactin levels greater than 215 ng/mL is almost always due to a prolactinoma, but it can reach this high in pregnancy. Also, she is not menstruating, which could still be normal after pregnancy, or she may be pregnant again. Hypothyroidism can cause an elevated prolactin level, but it never reaches 200 ng/mL. Primary empty-sella syndrome occurs with CSF leakage across the diaphragmatic sella, enlarging the sella so that it appears "empty." Empty sella syndrome typically does not cause abnormalities in pituitary function, but can cause pituitary hormone deficiencies. Prolactin levels can be mildly elevated, but remain <50 ng/mL.

A 58-year-old woman comes in for follow-up. She was diagnosed with DM 6 months ago by the cardiologist, who is following her for Stage 3 CHF (stable). She follows an AHA diet, takes her medications, and follows her walking program. Meds: Metformin: 500 mg qhs Blood glucose levels: Fasting: 100-140 Postprandial: 200-240 Bedtime: 100-120 HbA1c: 8.5% Creatinine: 1.5 mg/dL Spot urine albumin: Creatinine 20 BMI: 30.5 kg/m2 Which of the following is the most appropriate next step in patient care? A) Add pioglitazone to metformin. B) Increase metformin to 1,000 mg bid in steps as tolerated. C) Stop metformin and start either repaglinide or nateglinide. D) Stop metformin and start pioglitazone. E) Add acarbose or miglitol to metformin.

Answer C. Stop metformin and start either repaglinide or nateglinide. Explanation Expect this sort of question on your examination. Metformin contraindications and side effects are major topics. Because of this patient's creatinine (1.5), she should not have been started on metformin, according to the FDA-approved prescribing practices. Only the options that discuss stopping the metformin should be entertained as correct. All options that include adding a glitazone drug should be excluded because of this patient's history of Stage 3 CHF. This leaves us with "stop metformin and start either repaglinide or nateglinide." Giving special consideration to this patient's timing of hyperglycemia (postprandial), you should select the drug most suited to address the problem. Repaglinide and nateglinide are in the family of secretagogues that are designed to reduce postprandial hyperglycemia. Acarbose and miglitol are α-glucosidase inhibitors. These drugs slow postprandial glucose absorption and could be useful in this setting, but the option suggests adding them to metformin, which is inappropriate in the setting of a creatinine 1.5. Metformin absolute contraindications: Creatinine ≥ 1.5 in men Creatinine ≥ 1.4 in women The drug should be avoided in patients with renal, hepatic, or cardiac dysfunction; hypoxic states; alcohol abuse; and a prior history of lactic acidosis.

A 25-year-old female nursing assistant has a 2-month history of palpitations. She is trying to lose weight. On exam, her thyroid is not palpable, and she has a fine tremor. FT 4 2.1 (↑), TSH 0.01 (↓), thyroglobulin (↓) What is the most likely diagnosis? A) Hot nodule B) Subacute thyroiditis C) Surreptitious use of thyroid hormone D) Graves

Answer C. Surreptitious use of thyroid hormone Explanation This clinical picture is consistent with subacute thyroiditis or surreptitious use of thyroid hormone. The 2-month history suggests thyroiditis, but her gland is nontender, in which pain is almost always present in these patients. A hot nodule is not likely because there is no nodule noted on physical exam. The low thyroglobulin supports surreptitious use of thyroid hormone as thyroglobulin levels are elevated in Graves and with toxic nodules.

A 68-year-old woman has lost 15 pounds and has fatigue. There is a 2-cm nodule in the thyroid; pulse is 90; the skin is dry; and there is a mild tremor of the outstretched hands. A free T 4 is 1.9 (0.9-1.5) and TSH is < 0.05 (0.2-4.2). What is the next most appropriate step in evaluating the nodule? A) Ultrasound guided fine needle aspiration B) Measure TSH receptor antibodies C) Technetium scan D) Measure T 3

Answer C. Technetium scan Explanation This is an example of an elderly person with apathetic thyrotoxicosis lacking the classic symptoms of hyperthyroidism. The suppressed TSH suggests that the nodule is hyperfunctioning, and this should be confirmed using a technetium scan. Hot nodules rarely require FNA. As T 4 is elevated, an elevated T 3 will not be helpful diagnostically.

A 56-year-old man has been well all of his life but is now having episodes of crashing headache, sweating, irritability, and ashen pallor. During one such episode, coworkers cajole him into making an immediate emergency department visit while you are on duty. You find his blood pressure to be 228/136 mmHg. With further questioning, you learn that his aunt had a "tumor taken off her thyroid gland" and that an uncle had a "tumor removed from on top of one of his kidneys." You also notice that your patient has a neat row of small nodules along the leading edge of his protruded tongue. In selected family members, chromosomal analyses might very well lead to prophylactic removal of which of the following? A) Pituitary glands B) Kidneys C) Thyroid glands D) Parathyroid glands E) Adrenal glands

Answer C. Thyroid glands Explanation This is the autosomal dominant kindred of Multiple Endocrine Neoplasia (MEN) syndrome, type 2b or 3, with the characteristic combination of pheochromocytoma, medullary thyroid carcinoma, and diffuse gastrointestinal ganglioneuromatoses (as manifested grossly at the bedside by the tongue nodules). Medullary thyroid carcinoma (MTC) originates in the thyroid parafollicular C cells, those responsible for the secretion of thyrocalcitonin. It may be either sporadic or familial. Familial forms can be isolated or an expression of MEN kindred. Mutations of the RET proto-oncogene have been identified in the germline DNA of patients with familial MTC syndromes. Genetic testing can identify patients affected by MEN and familial MTC, allowing early diagnosis and possible cure. Total preventive thyroidectomy has been recommended in all carriers of RET genetic defects, even in families at risk with mutations of the 618 or 620 codon, because the penetrance of FMTC approaches 100%, and a 100% concordance between presence of the disease and gene carrier status. This procedure would therefore represent the only possibility of achieving a 100% cure in subjects affected by familial medullary thyroid carcinoma. The primary patient's pheochromocytoma is obvious and warrants careful preoperative treatment prior to surgical removal.

A 47-year-old man comes to see you after being told that he needs to see a physician. He is an admitted "health freak" and recently underwent a voluntary DXA scan for body composition determination. The test showed body fat content a little below average and a bone density that was 1.5 standard deviations below average for young men (Z-score -1.5). The radiologist reviewing the test told him that his bones were less dense than they should be and that he should see a physician. In your office, he admits to taking mega-doses of vitamins but is unable to recall which ones. He reports some mild bone and muscle pain, which he attributes to his long workouts. Your examination fails to detect anything abnormal. His blood pressure is normal. Laboratory tests are as follows: Na+ 140 mEq/L (135-143 mEq/L) K+ 4.9 mEq/L (3.5-5.0 mEq/L) Cl- 104 mEq/L (100-109 mEq/L) HCO3- 23 mEq/L (22-30 mEq/L) Urea nitrogen 10 mg/dL (8-18 mg/dL) Creatinine 0.8 mg/dL (0.6-1.2 mg/dL) Glucose 88 mg/dL (65-110 mg/dL) Albumin 4.5 g/dL (4.0-6.0 g/dL) Ca+2 10.9 mg/dL (8.5-10.5 mg/dL) Phosphorus 5.2 mg/dL (1.5-4.5 mg/dL) 25(OH)-vitamin D 63 µg/L (10-55 µg/L) 1,25(OH)2-vitamin D 19 ng/L (18-62 ng/L) Intact PTH 1 pg/mL (10-65 pg/mL) Which of the following might explain his hypercalcemia and osteopenia? A)Excess calcium in his diet B) Hypoparathyroidism C) Vitamin A intoxication D) Surreptitious use of thiazide diuretics E) Vitamin D intoxication

Answer C. Vitamin A intoxication Explanation This question requires you to know that hypercalcemia may be due to vitamin A intoxication. This patient has hypercalcemia and osteopenia, along with chronic ingestion of large doses of vitamins. The lack of an elevated level of 1,25(OH)2-vitamin D rules out an excess of this particular hormone, while an elevated level of 25(OH)-vitamin D indicates that he is ingesting an excess of this particular hormone. However, we are less interested in excess 25(OH)-vitamin D because it has limited activity and presents a problem only when at markedly high levels. His mild elevation of 25(OH)-vitamin D is too little to explain his hypercalcemia. The enzyme that converts 25(OH)-vitamin D into 1,25(OH)2-vitamin D (1α-hydroxylase) is regulated by PTH and will protect a person from developing high levels of 1,25(OH)2-vitamin D. Another hormone with a well-recognized role in calcium homeostasis is PTH. This patient's PTH level is appropriate in the setting of hypercalcemia. A low PTH would indicate hypoparathyroidism only in the setting of low calcium. Primary hypoparathyroidism is not a cause of hypercalcemia. Because his PTH is low, renal excretion of phosphate is not being stimulated and the level of phosphate is elevated. There is no reason to suspect thiazide diuretics. His blood pressure and volume status are normal, and his potassium is high-normal. There is no reason to attribute his hypercalcemia to thiazide diuretic use. Increased dietary calcium intake rarely causes hypercalcemia and would not explain the patient's osteopenia. The only other choice is vitamin A excess. Vitamin A excess is an uncommon cause of hypercalcemia, but it can be seen with chronically high ingestion of vitamin A, usually at levels 10-fold greater than the recommended daily allowance. For reasons unknown, high levels of vitamin A cause calcium resorption from bone. Vitamin A excess may also cause muscle and bone pain, hepatic dysfunction, dry skin, nausea, headache, fatigue, ataxia, alopecia, hyperlipidemia, and irritability.

A 55-year-old man with past history of hypertension and diabetes is admitted for abdominal pain. His home medications are lisinopril, amlodipine, metformin, and glipizide. Overnight the patient becomes diaphoretic, and his wife reports to the nurse that he has been very irritable since missing his dinner meal tray to undergo diagnostic imaging to evaluate his pain. He is on IV fluids of 1/2 normal saline. On exam, the patient's vital signs are: temperature 37° C, BP 140/87, HR 95, RR 22, and 98% O 2 saturation. He is difficult to arouse and does not verbally respond to questions. A bedside capillary blood glucose measurement is measured to be 45 mg/dL. What is the next best step in the management of this patient? A) 15 gm of oral carbohydrate replacement. B) Discontinue glipizide. C) 1 mg SC glucagon. D) 25 gm IV 50% dextrose. E) Discontinue metformin.

Answer D. 25 gm IV 50% dextrose. Explanation This patient has developed severe hypoglycemia because of the impaired consciousness. This necessitates rapid treatment and reversal of hypoglycemia. Both IV dextrose and SC glucagon would be effective therapies for this patient's hypoglycemia. IV dextrose would be the preferred option because of its more rapid onset of action compared to glucagon. Thus, glucagon is often reserved for patients who lack effective IV access, but he has an IV, and the dextrose can be given quite easily. In less severe patients who are able to follow commands, oral replacement of glucose is quick and can prevent worsening of hypoglycemia. Discontinuing this patient's glipizide would be important as sulfonylureas can precipitate hypoglycemia in a fasting patient. The decision to continue metformin as an inpatient also carries risks as well. However, the choice to discontinue either should be made after immediate treatment of the patient's hypoglycemia is initiated.

You are asked to see a woman recently admitted to the ICU with hypotension. She is unable to give a history, and no family members or friends are available. Witnesses report that she was sitting at the airport waiting for a connecting flight when she passed out. The person sitting next to her noticed that she became quite agitated when it was announced that the flight would be delayed for 6 hours, just before she passed out. There is no evidence of a pulmonary embolism. During your examination, you note that she is very thin, lacks axillary hair, has sparse pubic hair, and has extra pigmentation on her gums and buccal mucosa. The blood pressure was 40/palp with a heart rate of 130 when she first arrived, but is now 100/64 with a heart rate of 78 with saline running. Her temperature is 98.2° F; the cardiac rhythm is sinus; the routine chemistry tests show a hyperkalemic metabolic acidosis; and the renal status is compatible with prerenal azotemia. Which of the following should you do now? A) Begin hydrocortisone 100 mg IV every 8 hours and order an ACTH stimulation test. B) Measure cortisol and FT4 levels before beginning thyroxine and methylprednisolone. C) Begin hydrocortisone 100 mg IV every 8 hours. D) Begin dexamethasone 4 mg IV and order an ACTH stimulation test. E) Measure ACTH and TSH before beginning prednisone 60 mg and thyroxine 100 μg every day.

Answer D. Begin dexamethasone 4 mg IV and order an ACTH stimulation test. Explanation There is a strong suspicion that this patient has adrenal insufficiency and is having an adrenal crisis. She is hypovolemic with a hyperkalemic metabolic acidosis. The hypotension responded well to saline fluids. The thin body habitus and loss of axillary and pubic hair imply a chronic adrenal insufficiency. The flight delay may have caused stress for some reason that she was not able to cope with because of the adrenal insufficiency. This may have been what tipped her over the edge. Whether she really has an adrenal crisis or not is not the issue. It is more important to begin treatment than it is to prove your suspicion. The treatment is to give her intravenous glucocorticoids as soon as possible. But which glucocorticoid should you give? Remember, you do not yet have proof of the adrenal insufficiency. You can start treatment based on a suspicion, but you must also pursue a diagnosis. The diagnostic workup begins with an ACTH stimulation test. To do this, order a baseline serum cortisol level, administer 1 ampule of ACTH, and repeat the cortisol level 30 minutes later. The important thing to remember is that the assay for cortisol will cross-react with most other glucocorticoids and also pick up prednisone, methylprednisolone, and especially hydrocortisone since it is cortisol under a different name. If any of these are administered, how will you know what her endogenous production of cortisol really is? The solution is to administer a glucocorticoid that does not interfere with the cortisol assay, such as dexamethasone. She needs a glucocorticoid to save her life; begin dexamethasone immediately. If her adrenal function proves to be okay, you can easily stop the steroid. To check her adrenal function, check for her response to ACTH. If the cortisol level is greater than 20 μg/dL, her adrenal reserve is intact and you can stop the dexamethasone. If the ACTH stimulation test shows adrenal insufficiency, stop the dexamethasone and switch her to hydrocortisone. This is the actual cortisol molecule, and it has an advantage over the other glucocorticoids. Hydrocortisone (cortisol) is primarily a glucocorticoid, but it also has some mineralocorticoid activity that will help with her electrolytes. The other answers are incorrect because a glucocorticoid other than dexamethasone should not be used for the above reasons. Hypothyroidism is unlikely, so starting thyroxine is not indicated. Once her adrenal crisis has resolved, you can reevaluate her.

A 62-year-old man presents with muscle aches and fatigue. Over the last 1 month he has been having more pain with movement, especially in his thighs and upper arms. He has difficulty getting out of a chair without using his arms. He has never had similar symptoms in the past, and he denies oral ulcers, hair loss, skin rash, swollen, or painful joints. His past medical history is significant for coronary artery disease with 2 myocardial infarctions and multiple stents placed. He has a history of hypertension, diabetes, and hyperlipidemia His medications include atorvastatin, metoprolol, lisinopril, clopidogrel, metformin, aspirin, and gemfibrozil. On physical exam, his blood pressure is 120/78; pulse is 65. His exam is significant for muscle tenderness of his thighs and upper arms. He has no rash, joint swelling, or joint pain. Laboratory testing reveals a creatine kinase (CK) of 750 IU/L (normal is 60-400 IU/L) with a normal creatinine. You diagnose statin-induced myositis, and stop his atorvastatin. His CK normalizes; however, his LDL rises to 190. What is the next step in management of his hyperlipidemia? A) Restart atorvastatin, follow CK, and titrate dose. B) Start a bile-acid sequestrant. C) Begin ezetimibe. D) Begin pravastatin, follow CK, and titrate dose.

Answer D. Begin pravastatin, follow CK, and titrate dose. Explanation The patient's history of coronary artery disease necessitates aggressive control of his major risk factor, namely his elevated LDL cholesterol. The low potency statins, while not as effective at lowering LDL as newer high potency statins, will still lower LDL by 20-30% while having a lower risk of statin-induced myositis. Restarting atorvastatin is inappropriate as the risk of myositis is high, especially in conjunction with gemfibrozil. While both bile-acid sequestrants and ezetimibe lower LDL, their effect on reducing cardiovascular morbidity and mortality has not been conclusively shown. Another potential option would be to stop BOTH the atorvastatin and gemfibrozil, then eventually restart atorvastatin alone. As this option was not offered in the answer choices, the best answer choice remains to restart the pravastatin.

A 62-year-old woman comes to clinic with diarrhea for 3 weeks. The patient began having nonbloody diarrhea three weeks ago. She has 4-5 soft bowel movements per day. She has no abdominal pain, nausea, or vomiting. Over the same time, she has developed fatigue and has become anxious about work and her family. On review of systems, she complains of palpitations. She has a past medical history of hypertension and hyperlipidemia and coronary artery disease. Medications include aspirin, simvastatin, hydrochlorothiazide, and lisinopril. On physical exam her heart rate is 110; blood pressure is 148/89, and she is anxious appearing. Her heart exam shows tachycardia, and no murmurs, rubs, or gallops. Her lung and abdominal exam is unremarkable. She has a resting tremor, and her reflexes are brisk. Blood testing shows an undetectably low TSH and elevated T 4 and T 3 . What is the next step in the management of this patient? A) Order a radioactive iodine uptake scan. B) Order a thyroid-stimulating antibody panel. C) Refer to endocrinology. D) Begin propranolol.

Answer D. Begin propranolol. Explanation The patient is having significant symptoms from hyperthyroidism and has a history of coronary artery disease. Beta-blockade is indicated in the setting of coronary artery disease, hyperthyroidism, and tachycardia. In addition, beta-blockade can control the symptoms of thyrotoxicosis. A radioactive iodine scan is the appropriate diagnostic test, but the patient's symptoms should be controlled while waiting for the test. The workup and initial management of hyperthyroidism should be started prior to endocrinology referral. While Graves disease is the most likely etiology of her hyperthyroidism, the initial management remains, controlling the patient's symptoms

You are called to evaluate a 55-year-old insulin-dependent diabetic woman with acute mental status changes during her hospitalization for lower extremity cellulitis. The nurse reports that during her last vital sign check, the patient was unable to verbalize answers to questions and appeared to be very agitated. On exam, she is afebrile, BP 140/87, HR 95, RR 22, and 98% O2 saturation. General exam shows the patient to be diaphoretic and somewhat pale. Heart exam reveals RRR without murmur or gallop. Lungs clear. Left lower extremity is tender to palpation without fluctuance with an area of erythema that has not spread beyond the marked area drawn at the time of admission. What is the next best step in the management of this patient? A) 25 gm IV 50% dextrose B) Emergent surgical consultation C) ECG D) Capillary blood glucose E) 1 mg SC glucagon

Answer D. Capillary blood glucose Explanation The patient is experiencing classic hypoglycemia. Common symptoms include diaphoresis, psychiatric changes, and mildly elevated heart rate and blood pressure. Visual disturbances are also reported among patients with hypoglycemia. In the inpatient setting, confirming blood sugar with a capillary measurement is quick and provides information rapidly. Once confirmed, treatment can be initiated. Depending on the patient's ability to respond and/or ability to obtain IV access would dictate whether the optimal therapy would be oral glucose, IV glucose, or SC glucagon. Emergent surgical consultation would be a consideration if the patient's cellulitis was rapidly worsening, or if there were signs and symptoms of necrotizing fasciitis, but the failure of the erythema to spread beyond the marked areas and the lack of pain out of proportion to the exam make that consideration less likely. An ECG would be indicated if you thought the patient's symptoms were due to myocardial ischemia. However, a blood glucose would be more rapidly reversed and should be ruled out immediately before proceeding with additional diagnostic testing.

A 32-year-old was recently diagnosed with Addison disease. For 1 month she has taken 5 mg of prednisone daily and feels better. Her only other medication is levothyroxine. Labs today showed sodium 130, potassium 4.9, and CO 2 20. What should you do now? A) Increase dose of prednisone. B) Check cortisol and ACTH. C) Do adrenal CT scan. D) Change prednisone to hydrocortisone.

Answer D. Change prednisone to hydrocortisone. Explanation Patients with primary adrenal insufficiency need glucocorticoid and mineralocorticoid replacement. While she is symptomatically improved, her electrolytes suggest that she is mineralocorticoid deficient. Prednisone has minimal mineralocorticoid activity, and giving her a higher dose of prednisone will not fix the electrolytes. Hydrocortisone has both glucocorticoid and mineralocorticoid activity and is the best answer. No helpful information will be obtained from an adrenal CT or from measuring cortisol and ACTH.

An elderly man comes to your clinic at the insistence of his children. Ever since his wife died last year, he has been depressed and losing weight. He moved closer to his children after her death. He smokes 2 packs a day, having started smoking in 1942 while in the army. He is taking propranolol prescribed by his physician in his previous hometown, who was apparently treating him for mild hypertension. He talks to you easily, but doesn't seem very interested. His heart rate is 89, blood pressure 128/80, height 5'10", weight 160 lbs., and BMI 23.0 kg/m2. The physical examination is unremarkable. You order a routine metabolic profile and CBC. You also order a chest x-ray because of his extensive smoking history. He comes back for the results in two weeks, and you tell him that the labs and x-ray were normal. Which of the following would you do next? A) Order a total body CT scan. B) Discontinue his propranolol and begin treating his hypertension with a thiazide diuretic. C) Begin low-dose fluoxetine for his depression. D) Check TSH and FT4. E) Order a CT scan of the chest.

Answer D. Check TSH and FT4. Explanation This question requires you to recognize apathetic hyperthyroidism. This scenario is a classic example of apathetic hyperthyroidism. He has clues for hyperthyroidism, but they can easily be missed. Because of his depression, it is no surprise that he may have lost weight. He also has a high risk for lung cancer because of his smoking history, and cancer could be another explanation for weight loss. But your initial workup is negative. Now is the time to be more creative. He is not tachycardic, but he is on propranolol. Nevertheless, his heart rate is not as slow as you might expect in someone taking β-blockers. Weight loss, depression, and higher-than-expected heart rate should lead you to at least consider hyperthyroidism. Checking his TSH and FT4 would be the next step if not already checked. It is inappropriate to begin fluoxetine or any other drug for depression until you are sure of the diagnosis. It is also premature to order CT scans at this time. Finally, changing his blood pressure drug is okay, but you should first check his thyroid status.

A woman comes to your clinic complaining of galactorrhea. She is 55 years old with no significant past medical history except for an uneventful hysterectomy for fibroids at age 45. She is not on any hormone therapy, not involved in a sexual relationship, wears loose-fitting clothing, and denies nipple stimulation. You notice that she is very tired and moves slowly. She even seems to talk slowly. On physical examination, you can easily express thin, white fluid from both breasts. She has no breast masses or tenderness. Her skin is dry. A recent mammogram was unremarkable. You order a prolactin level and a CT of her pituitary. She returns for follow-up, and you find her unchanged. The prolactin was mildly elevated at 80 ng/mL (normal: 1.4-24.2 ng/mL), and she has a pituitary macroadenoma. Which of the following is the next best step in patient care? A) Because of her age, recheck the prolactin in 6-12 months. B) Refer her to a neurosurgeon. C) Start bromocriptine. D) Check TSH. E) Look for an unknown non-pituitary cancer.

Answer D. Check TSH. Explanation This patient has signs and symptoms of hypothyroidism. The next test that should be ordered is a TSH level, which is expected to be markedly increased. The obvious evidence for hypothyroidism includes feeling tired, moving slowly, talking slowly, and dry skin. But don't overlook the galactorrhea, pituitary mass, and mildly elevated prolactin level. At first glance, this can be mistaken for a prolactinoma, but a prolactinoma of this size (macroadenoma) should be associated with prolactin levels > 200 ng/mL. A prolactin level < 100 ng/mL should always prompt you to look elsewhere―in this case, the thyroid. Some other causes of a mildly elevated prolactin that have been ruled out include pregnancy and excessive nipple stimulation. The pituitary mass in this patient is most likely not a tumor. It is a pseudotumor composed of TSH-secreting cells (thyrotrophs), which have hypertrophied in response to TRH stimulation. TRH also stimulates the secretion of prolactin; hence the elevated levels. "Start bromocriptine" and "refer her to a neurosurgeon" are incorrect because it is important to rule out a thyrotroph pseudotumor first prior to any therapeutic intervention. It would be inappropriate to treat with bromocriptine or surgery. "Look for an unknown non-pituitary cancer" is incorrect because the described clinical scenario does not suggest any reason to look for a non-pituitary cancer. "Recheck the prolactin in 6-12 months" is incorrect because, although rechecking the prolactin level would be appropriate at some time in the future, her hypothyroidism must be addressed first.

A 28-year-old woman is evaluated for a 6-month history of amenorrhea. Prior to cessation of cycles, she was experiencing heavy bleeding. She had an uneventful labor and delivery 2 years earlier. She takes fluoxetine 40 mg daily. There is a family history of maternal Graves disease. She complains of some fatigue and occasional headaches. On physical exam, BP is 130/90 and clear fluid is expressible from her nipples. MRI of the sella is obtained and reveals a homogenous mass measuring 1.3 cm in height. Laboratory: Basal metabolic panel Normal LH 3.1 mIU/mL (1.9-12.5 mIU/mL) FSH 2.2 mIU/mL (2.5-10.2 mIU/mL) Estradiol 41 pg/mL (30-375 pg/mL) Prolactin 106 ng/mL (3-28 ng/mL) Which of the following is the next step in her management? A) Check a visual field examination. B) Refer the patient for neurosurgical consultation. C) Begin therapy with cabergoline 0.5 mg twice weekly. D) Check serum TSH. E) Check serum 17-OH-progesterone level.

Answer D. Check serum TSH. Explanation Key points: Primary hypothyroidism is an important cause of mild hyperprolactinemia and must be ruled out in all patients before proceeding with further evaluation, including MRI imaging. An enlarged pituitary gland may be seen in this setting, which will regress with levothyroxine replacement. This patient has 2 important clues to the possibility of primary hypothyroidism: a maternal family history of autoimmune thyroid disease and a history of menorrhagia. Anti-psychotic agents can often cause mild hyperprolactinemia, but not usually an SSRI such as fluoxetine. 17-OH-progesterone is a test used to seek a diagnosis of 21-hydroxylase deficiency, which is the most common form of congenital adrenal hyperplasia. Such patients would also have amenorrhea but would have hirsutism instead of galactorrhea. A 1.3 cm pituitary mass would not be expected to be large enough to compress the optic chiasm, and so visual field testing would have a very low yield. Medical therapy with cabergoline or surgical therapy is never indicated for patients with hyperprolactinemia until primary hypothyroidism has been excluded.

Jessica Simpleton is a 40-year-old woman who presents with proximal muscle weakness and depression. Your physician assistant saw her initially and ordered a series of tests after examining her last week. Physical Exam: BP 150/100, P 110, RR 20, Temp 99° F HEENT: Moon facies, acne Hirsutism Heart: RRR with I/VI systolic murmur Lungs: CTA Abdomen: Central obesity, prominent striae Extremities: Thin arms and legs Buffalo hump Supraclavicular fat pads noted Laboratory: 24-hour urine cortisol: 250 µg/day (normal 20-90 µg/day) 1-mg dexamethasone test: morning cortisol level after p.m. dose given: 18 µg/dL (normal < 5 µg/dL) 8-mg dexamethasone test: morning cortisol level after p.m. dose given: < 50% of baseline cortisol level What is the most likely etiology for her symptoms? A) High stress levels B) Hypothyroidism C) Pseudo-Cushing syndrome D) Cushing disease E) Ectopic ACTH production

Answer D. Cushing disease Explanation She has high 24-hour urinary cortisol, does not suppress with 1-mg suppression test, but does suppress with the high-dose dexamethasone test. Cushing disease or a pituitary source of excess cortisol is the only common etiology that will give these test results. Non-pituitary causes or ectopic ACTH are less common and usually not suppressed by the high-dose dexamethasone test (30% of ACTH-producing carcinoids are suppressible). Pseudo-Cushing syndrome or high stress levels would be suppressed by the low-dose suppression test.

A 50-year-old woman has gained 50 pounds and bruises easily. She has a buffalo hump, proximal muscle weakness, and pale striae on her abdomen. A 24-hr urine cortisol is 25 (< 50). Which of the following is correct? A) The next step is to measure ACTH. B) She likely has an adrenal adenoma. C) She likely has pituitary Cushing's. D) Cushing's has been excluded.

Answer D. Cushing's has been excluded. Explanation Even though she has clinical signs and symptoms of glucocorticoid excess, the screening test is negative, so she does not have Cushing's. There is no reason to measure ACTH or to do any other testing.

A 36-year-old woman has gained 20 pounds, has high blood pressure, and many bruises. Her BMI is 30. Her only medication is a BCP. A serum cortisol drawn at 8 a.m. after 1 mg of dex at 11 p.m. was 10 (↑). What should you do next? A) Do an 8 mg dexamethasone suppression test. B) No further therapy. C) Measure ACTH. D) Do a 24-hr urine cortisol.

Answer D. Do a 24-hr urine cortisol. Explanation She has clinical signs of Cushing's and had a screening test. Unfortunately, the 1 mg DST is not a good choice for screening as she is on a BCP. Because of the estrogen, the level of 10 is falsely high. The best screening test is a 24-hour urine cortisol. All of the other options should only be performed after the diagnosis of Cushing's is confirmed.

A mildly overweight 25-year-old man comes to your office because of an elevated glucose. He attended a health fair where his glucose was checked with a glucometer. His random glucose was found to be 220 mg/dL, and he was advised to have his glucose checked again. He reports more frequent urination and greater thirst than usual. The fasting glucose is 126 mg/dL. Metabolic panel otherwise is normal with a creatinine of 0.7 mg/dL. Which of the following should you tell him? A) He is not diabetic. B) He may have diabetes and needs to be rechecked again next year. C) He has impaired glucose tolerance and should be rechecked in 1 year. D) He has diabetes and must talk with a diabetes educator and dietitian to begin a diet, exercise program, and metformin. E) He has diabetes but doesn't need treatment until his glucose reaches 140 mg/dL.

Answer D. He has diabetes and must talk with a diabetes educator and dietitian to begin a diet, exercise program, and metformin. Explanation He fulfills 2 diagnostic criteria for diabetes. The full set of criteria include: 1) random glucose ≥ 200 mg/dL with symptoms (polyuria, polydipsia, or unexplained weight loss); 2) fasting glucose ≥ 126 mg/dL; and 3) 2-hour 75 g oral glucose tolerance test (OGTT) ≥ 200 mg/dL. Now that you have diagnosed him, you must next determine whether he has Type 1 or Type 2 diabetes. He most likely has Type 2, because he is overweight, and there is no mention of metabolic decompensation. You must now determine whether to treat and, if so, how. There is no question about it―you must intervene immediately. There is nothing to benefit from waiting. However, in the absence of metabolic decompensation and severe hyperglycemia, it is most prudent to begin with lifestyle changes. He must be educated on the importance of an exercise program, and he must learn how to eat a healthful diet. Current recommendations now include beginning metformin in all patients with their initial diagnosis of Type 2 diabetes. This is because of the difficulty in achieving and sustaining glycemic control and achieving significant weight loss.

A 16-year-old high school student is referred for evaluation of amenorrhea. She is an "A" student and has been active in school activities. She denies any other health problems. Physical Examination: Vital signs: BP 140/80, P 60, RR 18, Temp 98.7° F Height: 4' 4"; weight 130 lbs HEENT: Wears glasses; otherwise normal Neck: Possible short neck Heart: RRR with II-III/VI systolic murmur Lungs: CTA Breast: Tanner Stage I-II; widely spaced nipples Abdomen: +BS, soft, no hepatosplenomegaly Extremities: Short legs noted GU: Normal pubic hair distribution Based on your findings, what do you expect her LH, FSH levels to be? A) Her LH will be low, FSH high. B) It depends on what time of day you draw the levels. C) Her LH will be high, FSH low. D) Her LH will be high, FSH high. E) Her LH will be low, FSH low.

Answer D. Her LH will be high, FSH high. Explanation She has Turner syndrome: short stature, broad chest with widely spaced nipples, short neck (I didn't use the keyword "webbed," but that would have made it really easy), short legs, little breast development, and normal pubic hair. The gonadotropins, LH and FSH, are usually elevated continuously. Her murmur is likely aortic stenosis, and the hypertension she has is common.

A 17-year-old boy presents with a 1-year history of breast enlargement and no other complaints. His height is 71 inches, he weighs 170 pounds, his testes measure ~ 1 cm, the thyroid is not palpable, and there is bilateral palpable breast tissue. Testosterone is 190 (280-800), ß-HCG is negative, FSH is 30 (2-12), LH 25 (4-15), and estradiol is 10 (20-80). What is the most likely cause of his breast enlargement? A) Testicular tumor B) Prolactinoma C) Nonfunctioning pituitary tumor D) Klinefelter syndrome

Answer D. Klinefelter syndrome Explanation The high FSH and LH and the small testes support the diagnosis of Klinefelter's. A prolactinoma would lead to low gonadotropins. A testicular tumor would have elevated ß-HCG and elevated estradiol. A non-functioning pituitary tumor would likely have low LH and FSH along with CNS symptoms and hypopituitarism.

A patient of yours comes to see you and complains about being tired. She gave birth to a healthy baby about 6 months ago, and she tells you that the infant is doing fine. Her obstetrician reported to you that the pregnancy and delivery were uneventful. You ask your patient to tell you more, and she says that at first everything was perfect. She had plenty of energy to take care of the baby and to do all of the housework; she easily lost the weight that she had gained; and she had no problem staying up at night to feed the child. After about 1 month, she began to get tired. At first she was able to ignore it, but her fatigue steadily worsened until she could barely function. She can no longer keep up with the baby's needs or the housework. She is having difficulty with nursing. She blames herself for everything that's wrong and begins to cry. Which of the following should you do next? A) Refer her to a psychiatrist for evaluation. B) Tell her that the symptoms of her "postpartum blues" will soon pass. C) Begin stimulants. D) Order TSH and FT4, suspecting hypothyroidism due to postpartum thyroiditis. E) Begin antidepressants.

Answer D. Order TSH and FT4, suspecting hypothyroidism due to postpartum thyroiditis. Explanation Many young mothers will report postpartum depression, which will resolve on its own. However, some of these cases of postpartum depression may actually signify postpartum thyroiditis. The typical presentation is an uneventful pregnancy and delivery followed by an initial period of hyperthyroidism lasting for about 1 month, which is followed by a protracted period of hypothyroidism lasting for many months and sometimes for a year. The initial period of hyperthyroidism is easily tolerated. After all, the woman will have considerable energy and will lose weight more easily. However, she will soon develop hypothyroidism. Once you verify with TSH and FT4 that she is hypothyroid, begin treating her with thyroxine. Her hypothyroidism is not likely to be permanent, so follow her monthly and adjust the thyroxine dose as needed, ultimately discontinuing it.

Following testing for lipid levels, the results for a 28-year-old male are consistent with the diagnosis of type I hyperlipoproteinemia. Which of the following is the most likely consequence of this condition? A) Glomerulonephritis B) Cholecystitis C) Coronary vascular disease D) Pancreatitis

Answer D. Pancreatitis Explanation Hyperlipoproteinemia type I is manifested by markedly elevated triglycerides with the predominant elevation coming from increased chylomicrons. The impact on cardiovascular risk is variable but is significantly less than would be expected based on the level of elevation that is commonly seen in these patients. Several renal diseases can cause secondary hypertriglyceridemia, but elevated triglyceride levels do not directly lead to renal parenchymal disease. Pancreatitis is recognized to occur more frequently in patients with the marked triglyceride elevation seen in patients with type I hyperlipoproteinemia and, the higher the triglyceride levels, the greater the risk of developing pancreatitis. No direct relationship between hypertriglyceridemia and cholecystitis has been reported in this condition.

A 37-year-old woman, 3 months postpartum, presents with tremor, palpitations, and heat intolerance. She is no longer nursing. On physical exam, she has a BP of 160/60, P 120, and T 99° F. She has an enlarged thyroid on exam. Lab: TSH: < 0.03 (low), T4: 20 (high), T3RU: 40% (high) Which of the following tests would be most useful in planning therapy? A) CT scan B) Thyroglobulin C) Thyroid ultrasound D) Radioactive iodine uptake

Answer D. Radioactive iodine uptake Explanation This postpartum female has symptoms and lab work suggesting hyperthyroidism. The most common cause in this setting would be postpartum thyroiditis. The most definitive test to rule out the alternative diagnosis of Graves disease would be a 24-hour radioactive iodine uptake. Patients with Graves disease have a high 24-hour radioiodine uptake, whereas patients with acute or subacute thyroiditis have a very low radioiodine uptake. The radioiodine uptake test cannot be performed if she is nursing. In this instance, most would consider watchful waiting. If she has postpartum thyroiditis, her symptoms will improve; if she has Graves', she will worsen or not improve.

A 69-year-old man tells you he has leg pain just above the knee, which limits his hobby of gardening and prevents him from dancing. The pain has no aggravating or alleviating features. Physical exam finds Heberden nodes and crepitance without effusion or pain on motion in the knees. Neurologic exam is physiologic. His chemistry panel reveals normal liver enzymes and bilirubin, CBC, and calcium. An alkaline phosphatase is 3 times normal. Which of the following is the most likely diagnosis? A) Osteoarthritis B) Stress fracture C) Chondrocalcinosis D) Iron deficiency anemia E) Paget disease

Answer E. Paget disease Explanation Paget's causes an asymptomatic elevation of alkaline phosphatase most commonly. Since the patients can be asymptomatic or have aching that is similar in quality to the pain of DJD, indications for intervention are important to know: encroachment on neuro-foramen, CHF from increased blood flow, threat to weight-bearing bones, and pain are reasons for treatment.

A 65-year-old woman is evaluated after sustaining a fractured wrist. She underwent menopause at age 48 and never took estrogen replacement therapy. She has a history of mild hypertension controlled with HCTZ 25 mg/day. She does not smoke and takes no other medications except calcium carbonate 500 mg daily. Bone mineral density testing shows a T score of -2.7 lumbar spine and -3.1 femoral neck. Laboratory: Serum calcium 11.1 (8.6-10.4 mg/dL) Serum phosphorus 3.1 (2.5-4.5 mg/dL) iPTH 49 (10-65 pg/mL) TSH 0.5 (0.5-5.5 mU/L) 25(OH)-vitamin D 25.6 (20-100 ng/mL) Which of the following should be the next step in this patient's management? A) Treat the patient with vitamin D for 8 weeks and retest serum calcium and PTH. B) Check Free T4 level. C) Stop HCTZ and calcium tablets and repeat serum calcium in 4 weeks. D) Refer the patient for parathyroid surgery.

Answer D. Refer the patient for parathyroid surgery. Explanation Key points: Hypercalcemia and a normal (inappropriately normal) PTH level establish the diagnosis of primary hyperparathyroidism. Bone mineral density T score of worse than -2.5 is an indication for surgical referral in patients with primary hyperparathyroidism. Although hyperthyroidism can cause osteoporosis, a TSH of 0.5 is not low enough to support a diagnosis of hyperthyroidism (must be < 0.01). Although HCTZ can cause mild elevation in serum calcium, it will not cause osteoporosis, and most patients with HCTZ-associated hypercalcemia have underlying hyperparathyroidism. Although a 25(OH)-vitamin D level of 25.6 is somewhat low and could cause a secondary elevation in the PTH level, this scenario would not be associated with either hypercalcemia or with osteoporosis. Therapy with bisphosphonates is a 2nd line therapy for osteoporosis that is associated with primary hyperparathyroidism. Parathyroid surgery should be the first approach per current guidelines.

A 70-year-old woman with Type 2 diabetes and coronary artery disease has a hip fracture and is scheduled for surgery tomorrow. You are called by the anesthesiologist to evaluate her preoperative thyroid function tests. Her exam is remarkable for a B/P 130/80, pulse of 80, an easily palpable thyroid, and dry skin. Lab: TSH 18 (0.2-4.2), free T4 0.8 (0.9-1.5). What advice should you give? A) Give 400 µg IV levothyroxine and proceed with surgery. B) Give 25 µg IV triiodothyronine and proceed with surgery. C) Start 100 µg oral levothyroxine and delay surgery until thyroid function is normal. D) Start 25 µg of oral levothyroxine and proceed with surgery. E) Delay surgery and repeat TFTs in 4 weeks.

Answer D. Start 25 µg of oral levothyroxine and proceed with surgery. Explanation There is inadequate evidence to justify deferring scheduled surgery in patients with mild hypothyroidism (Arch Int Med 145:893, 1983). 400 µg of IV levothyroxine or 25 µg of T3 would be risky therapy in an elderly lady from a cardiovascular standpoint.

A 25-year-old woman comes to see you for irregular menses and difficulty getting pregnant. She denies unusual stress or any recent changes in her health. Her periods were regular until 8 months ago. She briskly walks 1 mile 4 days per week and has been dieting for 1 year. Her weight has decreased 47 pounds during the previous year. Your physical examination shows a small thyroid nodule and somewhat brisk deep tendon reflexes. You order a series of tests and have her return in 3 weeks to discuss the results, which are as follows: Na+ 140 mEq/L (135-143 mEq/L) K+ 4.0 mEq/L (3.5-5.0 mEq/L) Cl- 104 mEq/L (100-109 mEq/L) HCO3- 24 mEq/L (22-30 mEq/L) Urea nitrogen 9 mg/dL (8-18 mg/dL) Creatinine 0.8 mg/dL (0.6-1.2 mg/dL) Glucose 85 mg/dL (65-110 mg/dL) Ca+2 9.9 mg/dL (8.5-10.5 mg/dL FT4 1.9 ng/dL (0.7-1.5 ng/dL) TSH 0.01 mU/L (0.5-5.0 mU/L) Pelvic ultrasound was normal. Nuclear medicine thyroid scan showed mildly enlarged thyroid with generally diffuse uptake, except for a 1.5 cm cold nodule. Nuclear medicine thyroid uptake was 40% (normal 10-35%). Which of the following recommendations should be given to her at this time? A) PTU 100 mg every 8 hours with follow-up in 2 months. B) Begin oral contraceptives and repeat the thyroid tests in 6 months. C) Ultrasound-guided biopsy of her thyroid nodule. D) Surgical resection of her thyroid, followed by thyroxine replacement. E) Radioactive iodine ablation of the thyroid gland, avoid getting pregnant for at least 6 months, and biopsy the nodule if it still exists after the ablation.

Answer D. Surgical resection of her thyroid, followed by thyroxine replacement. Explanation She is hyperthyroid due to Graves disease but also has a cold nodule. The best treatment option in someone with Graves disease complicated by a cold nodule is surgical resection, followed by appropriate thyroxine replacement. This will take care of both the Graves disease and the cold nodule. Even though most cold nodules are benign, any cancer that may be present is most likely to occur in a cold nodule. Radioactive iodine ablation is an excellent treatment option for Graves disease, but it will not have any effect on the cold nodule because cold nodules do not take up iodine, by definition. Thus, after ablation her nodule will still be present and will have to be biopsied, but now the architecture is altered, and the biopsy and any necessary nodule resection may be more difficult. Surgical resection will also give this patient a faster return to euthyroidism and probable return to regular menses and fertility. Granted, this is a difficult question with several possible treatment plans. However, this scenario is one of the reasons to refer a patient with Graves disease for surgical resection. The vast majority of cases are treated with radioactive iodine ablation. PTU will address the hyperthyroidism, but will not address the cold nodule. An ultrasound-guided biopsy is not a bad choice, but you must still address the hyperthyroidism after the nodule has been taken care of. If the nodule is benign, then you can proceed to the ablation. If the nodule is not benign, then you will have to refer her for surgical resection anyway. Again, radioactive iodine ablation will address the Graves disease, but will not address the cold nodule. Waiting 6 months to examine the nodule could cause unacceptable advancement of any malignancy that may be present. Starting oral contraceptives will not address any of her problems. Remember: Any woman receiving a radioactive iodine ablation must avoid pregnancy for a minimum of 6 months. Also, since oral contraceptives are less effective in a woman who is hyperthyroid, she must use two forms of birth control or avoid sex altogether. The dose for a thyroid scan and uptake is considerably smaller, and no special precautions are required after the scan and uptake. Though the fetus is probably safe when exposed to the small dose used in scans, pregnant women should never receive radioactive iodine because of the potential threat to the fetus.

You are asked to see a 68-year-old woman who became dizzy and fell last week. She reports becoming shaky, sweaty, and slumping to the floor while shopping at about 11:30 a.m. She recovered after lying on the floor for several minutes and eating candy. She takes calcium and multivitamins. Past medical history includes a Billroth II for ulcers in 1962, C-section, and penicillin allergy. She had met a friend for breakfast at a donut shop that morning. Her usual breakfast consists of a boiled egg, brown toast and butter, and fruit. She generally eats a late-morning snack, and a meat-containing salad and hard roll for lunch at 1 p.m. She occasionally has similar, milder episodes, but in the past 10 years has had very few such episodes. She runs 3 miles three times weekly, does yoga daily, and works out with free weights twice weekly. She has no cardiac history, and her parents are still alive in their late 80s, as are their siblings. Physical exam reveals BP 116/78 and P 68. Musculature in upper arms and calves is well developed and delineated. Other than well-healed surgical scars on the abdomen, the exam is physiologic. The above is a prelude to asking you to identify "Whipple's triad." Which of the following comprise Whipple's triad? A) Symptoms of hypoglycemia, pancreatic lesion, and low serum glucose B) High serum glucose, symptom relief with insulin, and symptoms of hyperglycemia C) Symptoms of hypoglycemia, pancreatic lesion, and high serum glucose D) Symptoms of hypoglycemia, low serum glucose, and symptom relief with glucose E) Low serum glucose, abdominal pathology, and symptom relief with glucose

Answer D. Symptoms of hypoglycemia, low serum glucose, and symptom relief with glucose Explanation This patient scenario leads you to understand causes of hypoglycemia and to differentiate between fasting (insulinoma) and post-prandial (dumping syndrome and "reactive") hypoglycemia. Once you discriminate between hypoglycemia that occurs in the fasting state (due to insulin excess, either endogenous or exogenous) and hypoglycemia that occurs after eating, the correct diagnosis is post-gastrectomy syndrome. Also, it is necessary to remember that Whipple's triad of hypoglycemic symptoms occurs during periods of low serum glucose and is relieved by a source of sugar.

A 60-year-old diabetic woman presents to your clinic as a referral from your favorite orthopedist following a wrist fracture in which he found a serum calcium of 10.9 mg/dL. She reports that she has been having "bone pain" in her hips and knees for several years. Looking over her records, you see that she has been on dialysis for 5 years or so. Which of the following is most likely true? A) Hypercalcemia in this setting is usually acute and life-threatening. B) Aluminum salts are contraindicated. C) Parathyroid hormone half-life is decreased. D) The patient likely has tertiary hyperparathyroidism. E) Osteitis fibrosa is unlikely.

Answer D. The patient likely has tertiary hyperparathyroidism. Explanation This is seen commonly in renal patients where the parathyroid glands become autonomous. The hypercalcemia develops gradually over time and is rarely life-threatening. It is associated with osteitis fibrosa. It can be treated with aluminum "binders." Finally, parathyroid hormone half-life is actually increased in renal insufficiency.

A 38-year-old woman is referred to you by a neurologist who was seeing her for severe headaches. His final diagnosis was stress headaches, but because a head MRI showed an empty sella, the neurologist wanted you to see her. On questioning, you learn that she has 7 children and all are healthy. When the first child was born, she experienced a difficult delivery and considerable bleeding that required a transfusion. All subsequent deliveries were uneventful. The review of systems was otherwise negative. Physical examination shows normally pigmented abdominal striae. The blood pressure is 100/60. You order some blood tests and find that the electrolytes, glucose, and TSH are normal. Which of the following is the most correct thing for you to do next? A) Diagnose Sheehan syndrome and begin replacement therapy. B) Evaluate for pituitary hypofunctioning. C) Evaluate for pituitary hyperfunctioning. D) Refer her to a neurosurgeon for exploration of her pituitary gland. E) Tell her that everything is normal and discharge her from your clinic.

Answer E. Tell her that everything is normal and discharge her from your clinic. Explanation The important point to remember for this question is that an empty sella may be normal. This patient does not present any indication of pituitary dysfunction. The history of blood loss at the time of delivery may suggest Sheehan syndrome (hemorrhagic necrosis of the pituitary), but one of the most sensitive indicators of pituitary dysfunction is ovulation. This patient was able to get pregnant 6 more times; the hypothalamic-pituitary-ovarian axis must be working properly. She does not have Sheehan syndrome, but her obstetrical history offers another clue. Her 7 pregnancies have most likely caused her empty sella. With pregnancy, the pituitary enlarges and has a greater blood flow. This produces an upward pressure on the diaphragm covering the top of the sella, which stretches and expands the diaphragm. After pregnancy, the pituitary returns to a normal size with a normal blood flow, but the diaphragm is acellular and remains stretched. With each subsequent pregnancy, the diaphragm becomes progressively more stretched. This allows the diaphragm to bulge downward and into the sella under the pressure of the CSF fluid above it. In some cases, the diaphragm has become so loose that the CSF fluid displaces the majority of the pituitary to the periphery of the sella. A CT of the head will fail to identify pituitary tissue, and the term "empty sella" will be used. In actuality, the sella is not empty, but contains CSF fluid and normal pituitary tissue that has been pushed to the periphery. The pituitary tissue can work perfectly normally in this environment. It is inappropriate to refer her to a neurosurgeon. Pituitary exploration may disrupt the normal functioning of her existing pituitary tissue and result in the development of hypopituitarism. While there is no evidence of pituitary dysfunction, the abdominal striae may trick some people to suspect Cushing syndrome, but there is no mention of hyperglycemia, hypokalemia, or the typical body habitus. Stretch marks are a common occurrence, especially following pregnancy. The typical stretch marks associated with Cushing syndrome are broad and appear purplish from the coloration of underlying tissue.

You are asked to see a patient in the surgical ICU who is doing poorly 7 days after emergency repair of a perforated colon. He is 68 years old and developed mesenteric ischemia, which resulted in his perforation. He is now septic and requires ventilator support. You note numerous laboratory derangements, including abnormal WBC, Hct, HCO3, creatinine, hepatic enzymes, and arterial pH. He also has a mildly depressed TSH and FT4. He is receiving adequate and appropriate care in the ICU, but the surgeon wants your opinion about his thyroid dysfunction. Which of the following should be your response? A) The patient has mild hyperthyroidism that doesn't need treatment. B) The patient has mild hypothyroidism that doesn't need treatment. C) The patient has mild hypothyroidism and should start thyroxine therapy. D) The patient probably does not have any thyroid problems. E) The patient has mild pituitary dysfunction and should start thyroxine therapy.

Answer D. The patient probably does not have any thyroid problems. Explanation This patient is critically sick. In severe illness, the pituitary secretes less TSH, which results in a low or low-normal FT4. However, T4 is not the active hormone; T3 is active. In severe illness, T3 levels are greatly suppressed. With the T4 being lower, there is less T4 available to be converted into T3. However, there is still considerable T4 present. In order to protect the body from the active T3 hormone, the T4 is converted into an inactive hormone called reverse T3 (rT3). Levels of rT3 are rarely measured, but if it was measured in this patient, it would be greatly elevated. This patient is most likely euthyroid, but also very sick. He does not have hypothyroidism or hyperthyroidism, either primary or secondary. There is no need to act on his abnormal thyroid tests; instead treat his underlying illness.

Your patient is a 45-year-old male whose father died of an MI at age 57. He does not smoke and is not on any medication. BP: 120/72 HDL: 42 mg/dL What is his target LDL? A) < 130 B) < 190 C) < 70 D) < 100 E) < 160

Answer E. < 160 Explanation Recall the following NCEP-III guidelines from the NCEP Update report 2004: Stratification LDL Goal Drugs CHD/CHD risk equivalents < 100 ≥ 100 2+ RF & 10Y risk 10-20% < 130 ≥ 130 2+ RF & 10Y risk < 10% < 130 ≥ 160 0-1 RF < 160 ≥ 190 NCEP-III risk factors: Age: males ≥ 45 and females ≥ 55 Family history of premature CHD in 1 st degree relatives (males < 55, females < 65) Current cigarette smoking Hypertension (≥ 140/90 or on medication) HDL ≤ 40 (HDL ≥ 60 = negative RF) CHD equivalents - PAD - AAA - TIA - CVA - Carotid stenosis - DM So our patient just has one risk factor (his age of 45). Therefore his LDL goal is < 160.

A patient of yours comes to your office for a routine follow-up. He is 50 years old and has had Type 2 diabetes for 8 years. He is mildly overweight with a BMI of 28, has lost a few pounds over the past 4 months, is watching his diet as best he can, and is walking 1-2 miles every evening after work. You have been treating him with a thiazolidinedione, a statin, and a baby aspirin every day for the past 3 years. Three days before coming to your office he went to the lab for the routine tests that you asked him to get. Your physical examination is unremarkable. His blood pressure at the last visit was 132/80 and today is 134/82. His fasting LDL is 98 mg/dL; the liver function tests, including ALT, are well within normal limits; electrolytes are all normal; the creatinine is 0.8 mg/dL; the random urine microalbuminuria screen is 180 μg/mg. Which of the following is the best next step in care? A) Start a low-dose diuretic such as hydrochlorothiazide 12.5 mg every morning. B) Tell him to keep up the good work and continue to lose weight C) Increase the statin. D) Add a calcium channel blocker for his hypertension. E) Add an ACE inhibitor or an angiotensin-receptor blocker (ARB).

Answer E. Add an ACE inhibitor or an angiotensin-receptor blocker (ARB). Explanation The random urine screen indicates microalbuminuria. Normal albumin excretion is < 30 mg/day, microalbuminuria is 30-300 mg/day, and macroalbuminuria is > 300 mg/day. Collecting an accurate 24-hour urine is difficult and problematic. Fortunately, creatinine excretion is somewhat consistent when adjusted for standard body surface area. This has led to use of the albumin:creatinine ratio in a random spot urine as a surrogate marker. A normal ratio is < 30 μg/mg, microalbuminuria is 30-300 μg/mg, and macroalbuminuria is > 300 μg/mg. If the ratio is only slightly above normal, it is best to repeat the random urine or to ask for a 24-hour urine to better quantitate the albumin excretion. In addition, any urinary tract infection must be completely resolved before checking for albuminuria. His ratio is well within the range of microalbuminuria, and there is no suspicion of a urinary tract infection. Once a diabetic has microalbuminuria, the patient must be treated. The goal of treatment is to slow the progression of the nephropathy in an attempt to prevent end-stage renal disease. Several studies have shown that an ACE inhibitor is an excellent drug for this purpose, and some recent studies have shown that ARBs may be just as beneficial. The best course of action for this patient is to begin one of these drugs. The patient is certainly doing a good job by watching his diet, consistently walking, and losing weight. He should be praised for this, but he also needs treatment for the nephropathy. There is no reason to increase his statin, since the LDL is less than 100 mg/dL (goal). A calcium-channel blocker may be useful for hypertension, but is not the initial drug of choice in diabetics with nephropathy. Likewise, a diuretic can be useful for blood pressure control, but is not the i nitial drug of choice in a diabetic with nephropathy

An elderly man with benign prostatic hyperplasia (BPH) is scheduled for a TURP. He has complete obstruction and required a suprapubic catheter. He now has post-obstruction diuresis and a concentrating defect. The surgeon is having difficulty managing the patient's fluid status and electrolytes. He wants to perform the TURP as soon as possible and has consulted you for medical clearance. During your examination, you find a small goiter, tachycardia, and A-fib, as well as prostatic hypertrophy, a suprapubic catheter, and a large volume of dilute urine, but the urine osmolality is improving. Because of the tachyarrhythmia, you check his thyroid function. The lab reports TSH < 0.01 mU/L (normal 0.5-5.0 mU/L) and FT4 2.4 ng/dL (normal 0.7-1.5 ng/dL). The patient does not have anyone at home to help him and cannot go home with a suprapubic catheter. He has no insurance and cannot afford home health care. Your consult includes multiple recommendations, but which of the following is not included? A) Postpone surgery until his FT4 is within normal limits. B) Begin propylthiouracil. C)Begin β-blocker as needed to control symptoms of hyperthyroidism. D) Start unfractionated heparin. E) Begin non-radioactive iodine (Lugol's solution) immediately.

Answer E. Begin non-radioactive iodine (Lugol's solution) immediately. Explanation The patient is clearly hyperthyroid based on clinical symptoms and biochemical testing. It is unclear if he has Graves' disease versus a toxic multinodular goiter as a radioactive uptake scan is not available at this time. Iodine preparations such as Lugol's solution and potassium iodide cause immediate inhibition of preformed thyroid hormone secretion. These iodine preparation are not indicated in this situation as should he have a toxic multinodular goiter, the iodine would provide addition substrate for new hormone synthesis, potentially worsening any hyperthyroidism. Should Lugol's solution be given, it is only recommended at least one hour AFTER thionamide (such as PTU) administration. Elective surgery should ideally be postponed to correct hyperthyroidism in order to prevent the precipitation of thyroid storm. Though his surgery can be postponed for a short period of time to give the thyroid treatments a chance to start working, he needs the surgery soon. Sending him home with a suprapubic catheter until he is euthyroid is not a good option in this case. Therefore, your best strategy is to address his hyperthyroidism in anticipation of immediate surgery. PTU will decrease synthesis of new thyroid hormone, with the goal of decreasing hyperthyroidism in the post-operative period as it takes a few days to start working. A β-blocker will help control his symptoms of hyperthyroidism. A good choice would be a β-blocker that can be given IV and rapidly titrated during surgery, such as esmolol. Because he has A-fib, he must be suspected of having an atrial thrombus; therefore, he should be anticoagulated because he may revert to a normal rhythm once he becomes euthyroid. This can be done with a heparin product before surgery, which can be stopped preoperatively, and with warfarin after surgery.

A new patient is seen in your clinic complaining of palpitations, tremors, heat intolerance, weight loss, hair loss, three bowel movements every day, and fatigue. He has a mild goiter that is nontender, smooth, symmetrical, and without nodules. There is no cervical lymphadenopathy. Feeling confident about diagnosing Graves disease, you prescribe a β-blocker for his symptoms and order TSH, FT4, and a thyroid scan and uptake. He returns in 2 weeks. His symptoms are much improved since taking the β-blocker, but he is still clinically hyperthyroid. The TSH is < 0.01 mU/L (0.5-5.0 mU/L) and FT4 is 2.5 ng/dL (0.7-1.5 ng/dL). The thyroid scan reveals a symmetrical goiter that is homogeneous. The thyroid uptake is 30% (10-35%). What is most likely to explain his normal uptake? A) Use of expired radioactive iodine by the nuclear medicine technician B) Low dietary intake of iodine C) Hashimoto thyroiditis D) Presence of multiple hot nodules E) High dietary intake of iodine

Answer E. High dietary intake of iodine Explanation This question requires you to know what factors influence a radioactive iodine uptake test. A high uptake (> 35%) signifies a greater than usual uptake of radioactive iodine by the thyroid. The most common cause of high uptake in this country is Graves disease. Other causes of a high uptake include iodine deficiency (common in other parts of the world), hot nodules (if hot enough), toxic multinodular goiter (sometimes), high levels of β-HCG (it resembles TSH and can activate TSH receptors when present in very high concentrations such as seen with molar pregnancy, choriocarcinoma, and some cases of germ-cell tumors), and a pituitary tumor secreting TSH. A low uptake (< 5%) signifies a less-than-usual uptake of radioactive iodine by the thyroid. Causes of a low uptake include Hashimoto thyroiditis (most common), other causes of thyroiditis (e.g., subacute, postpartum, radiation, and amiodarone-induced), thyroid-suppressive drugs (e.g., propylthiouracil, methimazole, and lithium), iodine excess, ectopic thyroid tissue (struma ovarii), lack of a thyroid gland (radioablation or surgical removal), and expired radioactive iodine (extremely unlikely because of several quality assurance steps). The patient in this question probably has Graves disease because of an elevated FT4 in the face of an undetectable TSH and a symmetrical goiter with homogeneous uptake. The uptake in Graves disease is normally greater than 35%. His thyroid is undoubtedly taking up iodine at a much higher rate than normal. However, the thyroid gland does not discriminate between radioactive iodine and non-radioactive iodine (cold iodine). Some patients take iodine supplements (e.g., kelp pills), and this excess cold iodine competes with the radioactive iodine. Thus, less radioactive iodine is taken up by the thyroid while greater amounts of cold iodine are being taken up. This results in a lower uptake than expected. High intake of iodine can be verified by checking a 24-hour urine for iodine. The other choices are incorrect. Hyperthyroid Hashimoto thyroiditis (aka hashitoxicosis) can present with hyperthyroidism, symmetrical goiter, and suppressed TSH, but the uptake is low (< 5%). His uptake is too high for thyroiditis. Expired radioactive iodine is extremely unlikely. Low dietary intake of iodine would enhance the uptake, not suppress it. His scan is homogeneous, which argues against hot nodules.

A 79-year-old woman comes to you accompanied by her daughter visiting from out of town. The patient insists she is fine. The daughter confirms the patient's assertion that there's been no difficulty with swallowing or breathing, nor has there been a change in her mother's voice. The family reports weight loss, irritability, and anxiousness. P 104, regular. She has tremor of her outstretched arm and an irregularly textured thyroid. The trachea is not deviated. She cannot resist your pressure against raising her leg from the exam table. T4 is 17 and TSH < 0.1 mIU/mL. Which of the following is the most appropriate evaluation? A) CT neck B) MRI of head C) Thyroid ultrasound D) Thyroid scan E) Thyroid scan and uptake

Answer E. Thyroid scan and uptake Explanation You need to know thyroid function, not just its anatomy. The absence of symptoms due to mass effect from the thyroid—e.g., no recurrent laryngeal nerve impingement (i.e., no voice change, weakness, or "gravelly" sound), and no dysphagia or dyspnea—excludes the need for anatomic studies: MRI, CT, ultrasound, and scan. By obtaining a thyroid scan and uptake (which reveal the gland's functional ability to concentrate 131I), you will find diminished uptake except in the left lower pole of the thyroid where there is a "hot" nodule.

A 25-year-old man presents because he and his wife have been unable to conceive. He denies any medical problems and has been healthy most of his life. Past Medical History: Negative Social History: Works as a ski instructor in the winter; Works as a tennis instructor in the summer Family History: Unknown, was adopted Review of Systems: Has noted that his sense of smell doesn't seem to be as good as that of other people's. His wife gets upset with him because he doesn't notice her perfume. Physical Examination: 6'4" tall, weight 180 lbs. Body Habitus: Eunuchoid proportioned—His lower body segment > upper body segment. His arm span is 7 cm more than his height. GU exam: Testes are small for age, only 3 cm Which of the following is true with regard to his diagnosis? A) A karyotype will be diagnostic. B) If GnRH is given, FSH and LH will respond appropriately. C) Testosterone levels will be undetectable. D) His mental development is abnormally high for this diagnosis. E) His basal FSH and LH levels will be low or undetectable.

Answer E. His basal FSH and LH levels will be low or undetectable. Explanation He has Kallmann syndrome. These men are eunuchoid but also have increased arm spans, unlike patients with Klinefelter's. Anosmia or hyposmia is classic, and Kallmann syndrome patients have a normal karyotype (46,XY). Their testes are small but larger than those with Klinefelter's. They have basal FSH and LH levels that are low or undetectable, and these 2 do not respond to GnRH. Their testosterone is in the middle of the female range but certainly not undetectable.

A 24-year-old man with hypocalcemia is referred to you. He has a history of alcohol abuse and poor nutrition during his bouts of drinking. Prior episodes of hypomagnesemia were all related to bouts of alcohol abuse and spontaneously resolved within two weeks of abstinence. The patient's primary care provider noticed the hypocalcemia this time and has not been able to explain it. The patient reports fatigue and muscle cramps. You are not able to elicit Chvostek or Trousseau signs. You puzzle over the pertinent lab results until you realize the explanation for hypocalcemia is staring you in the face. Na+ 140 mEq/L (135-143 mEq/L) K+ 4.2 mEq/L (3.5-5.0 mEq/L) Cl- 104 mEq/L (100-109 mEq/L) HCO3- 24 mEq/L (22-30 mEq/L) Urea nitrogen 10 mg/dL (8-18 mg/dL) Creatinine 0.8 mg/dL (0.6-1.2 mg/dL) Glucose 88 mg/dL (65-110 mg/dL) Albumin 4.5 g/dL (4.0-6.0 g/dL) Ca+2 7.9 mg/dL (8.5-10.5 mg/dL) Phosphorus 4.9 mg/dL (1.5-4.5 mg/dL) Mg+2 0.5 mg/dL (1.4-2.5 mg/dL) 25(OH)-Vitamin D 40 µg/L (10-55 µg/L) 1,25(OH)2-Vitamin D 13 ng/L (18-62 ng/L) Intact PTH 6 pg/mL (10-65 pg/mL) As expected, all of the above labs normalize within one week of abstaining from alcohol and taking magnesium supplements. What was the most likely cause of this patient's hypocalcemia? A) Familial hypocalciuric hypocalcemia B) Pseudohypoparathyroidism C) Pseudohypocalcemia D) Barter syndrome E) Hypoparathyroidism due to hypomagnesemia

Answer E. Hypoparathyroidism due to hypomagnesemia Explanation This question requires you to know that the parathyroid cells require magnesium to secrete parathyroid hormone. Calcium is generally required for exocytosis, so cellular secretion is impaired by low levels of calcium. However, the state of hypocalcemia is the very state when the parathyroid cells need to be most active in secreting parathyroid hormone. Fortunately, the parathyroid cells utilize another divalent cation, magnesium, for exocytosis. However, this causes PTH secretion to decrease with hypomagnesemia. The normal albumin level eliminates pseudohypocalcemia. Barter syndrome is characterized by hypomagnesemia, hypokalemia, metabolic alkalosis, and hyperreninemic hyperaldosteronism. Pseudohypoparathyroidism refers to a syndrome caused by an abnormal PTH-receptor and presents with low calcium, high phosphorus, and high PTH levels. The patient in this vignette has a low PTH level. Familial hypocalciuric hypocalcemia does not exist. The syndrome being alluded to is familial hypocalciuric hypercalcemia.

A 32-year-old man comes to see you because his wife cannot conceive. He reports normal development, was taller than his peers, and he's had no change in sexual interest or activity. BP 112/68, P 88, Wt 220, Ht 71", Span 72" He has 2 cm of glandular tissue under the areola, minimally excess abdominal adipose, and testes 2 x 1 cm. His scrotum has rugae and terminal hair. Which of the following is the most likely diagnosis? A) Prolactinoma B) Hereditary small gonad syndrome C) Kallmann syndrome D) Pituitary adenoma E) Klinefelter syndrome

Answer E. Klinefelter syndrome Explanation This exam points toward eunuchoid development with span greater than height, which is compatible with late epiphyseal closure due to underproduction of testosterone; and the gynecomastia also points toward relatively insufficient testosterone production. The key is the small (usually described as firm) testes and gynecomastia, which point to Klinefelter syndrome. Kallmann syndrome is not correct, because congenital hypogonadotropic hypogonadism has low amounts of both estrogens and testosterone, so gynecomastia does not develop. Pituitary adenoma and prolactinoma do not have the skeletal manifestations listed. Hereditary small gonad syndrome is not a true syndrome.

A 25-year-old woman presents with a 3-week history of palpitations, anxiety, fatigue, and difficulty sleeping. She notes a 5-pound weight loss. She denies recent viral infections, fever, or change in her vision. Her mother is receiving thyroid replacement. On examination, BP 110/76, pulse 104, the thyroid gland is enlarged twice normal on the right and three times normal on the left and is not tender. There is no exophthalmos. Laboratory: CBC normal ESR 31 TSH < 0.01 mIU/L (0.5-5.0 mIU/L) Free T4 2.8 ng/dL (0.7-1.6 ng/dL) Which of these possible next steps would be most helpful in her subsequent management? A) Begin therapy with methimazole. B) Measure anti-thyroid peroxidase antibodies. C) Measure serum free T3 level. D)Perform thyroid ultrasound. E) Measure 24-hour thyroid uptake of 131I.

Answer E. Measure 24-hour thyroid uptake of 131I. Explanation Key points: The primary differential diagnosis for this patient is between Graves disease without eye involvement and non-painful thyroiditis (lymphocytic). The only test that will distinguish between these is the 24-hour 131I uptake. The short duration of symptoms mandates that subacute thyroiditis be strongly included in the differential diagnosis. The absence of a tender thyroid gland, fever, or a very high ESR rules out the possibility of de Quervain thyroiditis (granulomatous), but the nontender lymphocytic variant is possible. In thyroiditis, the 24-hour 131I uptake will be very low, whereas in Graves disease, it will be elevated. A thyroid ultrasound is not likely to yield any important new information. The goiter of Graves or thyroiditis can often be asymmetrical. Anti-thyroid antibodies, such as thyroid peroxidase, can be elevated in both Graves and in thyroiditis. Once the TSH is suppressed and the free T4 is found to be elevated, no further use comes from measuring the free T3. This would only be measured if the TSH were suppressed with a normal free T4. It would not be appropriate to begin therapy with methimazole until the diagnosis of Graves is proven with an elevated 24-hour 131I uptake. Methimazole is unhelpful and incorrect to give to a patient with thyroiditis.

A 78-year-old woman was last seen for her annual Pap smear 1 year ago. At that time she was fatigued and had several yeast infections, which she treated with OTC preparations. She continues to be fatigued, has nocturia twice nightly, and has had several yeast infections. She takes no medications and occasionally takes calcium. Her mother died of an MI, her aunt of a stroke, and her uncle was on dialysis due to diabetes before his death due to myocardial infarction. You find BP 146/94; P 84, regular; Wt 187 lbs.; Ht 65.5". There is no retinopathy, no bruit; she has decreased sensation to fine touch. FSBG (fasting serum blood glucose) 238 mg/dL, cholesterol 287 mg/dL, triglycerides 681 mg/dL After receiving these numbers, you start her on diet, exercise, and an ACE inhibitor for her hypertension. She returns in 3 months with the following values: FSBG is 189 mg/dL, cholesterol 224 mg/dL, triglycerides 187 mg/dL, weight 182 lbs, and BP 122/78. Her creatinine is 0.7 mg/dL. At this point, which of the following is the next appropriate treatment? A) Rosiglitazone B) Glyburide C) NPH insulin D) Nothing E) Metformin

Answer E. Metformin Explanation For a consistently elevated fasting glucose with diet modification, metformin is indicated to diminish hepatic gluconeogenesis. It is the first-step treatment for Type 2 diabetes. She now has had 2 fasting blood glucoses > 126 mg/dL, and you have made the diagnosis of Type 2 diabetes. She should be started on metformin now if there are no contraindications (most important to know is that for women, creatinine must be < 1.4 mg/dL and for men < 1.5 mg/dL).

A 35-year-old man presents for a pre-work physical. He is normal, except that you note he has extremely small testicles and prominent gynecomastia. He says that no one else in his family has a similar body type to himself. Laboratory: GnRH: High FSH: High LH: High Ferritin: Normal Prolactin level: Normal What is his karyotype? A) XXXXXXXXXy B) XY C) XX D) XYY E) XXY

Answer E. XXY Explanation He has Klinefelter syndrome, which has karyotype XXY. The GnRH, LH and FSH are all elevated because his testicles won't respond to them, and he has no feedback inhibition of these. Patients with secondary hypogonadism due to an abnormal hypothalamic pituitary axis have a low GnRH, FSH and LH with a low testosterone. You would see this in prolactinoma and Kallmann syndrome (hypogonadotropic hypogonadism). Note that with Kallmann syndrome, you may find a patient with "smell difficulties" and sometimes a cleft palate—and it runs in male members of the families.

A 75-year-old female has been taking 1 grain (60 mg) of Thyroid-S Armour® thyroid extract daily for 15 years and feels great. She won't take any other type of thyroid medication. Today her free T 4 is 0.5 (0.9-1.7), TSH 1.2 (0.2-4.2), and T 3 2.3 (0.8-2). Her exam (including the thyroid exam) is normal. Which of the following should you recommend? A) Increase dose to 1½ grains. B) Make no changes. C) Measure free T 3 . D) Measure antithyroid antibodies.

B. Make no changes. Explanation This clinical scenario points out the difficulty of using combination T 4 /T 3 preparations. Because there is no way of knowing exactly how much T 4 and T 3 are present, these levels will fluctuate and vary from batch to batch. Nevertheless her TSH is normal, indicating that her current replacement dose is adequate. Measuring T 3 and antithyroid antibodies does not aid in making a treatment decision. Increasing the dose is not the right answer as her TSH is normal.

A 42-year-old woman G2P2 had menarche at age 13 followed by regular q 27-day menses with 4 days flow in the ensuing years, until 9 months ago. She also reports increased pigmented facial hair and some scalp hair thinning. BP 138/84 Facial and trunk acne is present Temporalis hair recession Deep voice Pigmented hair on chin, chest, shoulders, extremities, and gluteal region Laboratory is ordered and the results show: DHEA-S extremely high (1183), testosterone high (371 ng/dL), LH low (2.7), FSH low (2.2), estradiol low (31) Which of the following is the most likely diagnosis? A) Hyperthyroidism B) Hypogonadotrophic hypogonadism C) Adrenal cancer D) PCOS (polycystic ovary syndrome) E) Menopause

C. Adrenal cancer Explanation This question asks you to know that 2 things―late onset and rapid progression of androgenization―mean cancer. The ovary is the major source of measured testosterone in the woman; the adrenal is the major producer of DHEA, so extraordinarily high DHEA-S means adrenal cancer. To evaluate hirsutism requires DHEA-S and testosterone. If the hirsutism is milder and insidious with symptoms beginning in puberty or the early 20s, it is more likely due to PCOS; in which case, LH: FSH ratio over 2.5 along with high normal or above normal testosterone occurs. Two unfortunate things about treatment for adrenal carcinoma: it demands prompt excision, and it has a poor prognosis.

A 47-year-old woman comes to your clinic complaining of night sweats and marked mood swings. She says that she is experiencing the same unbearable symptoms of menopause that her mother experienced and she wants relief. She routinely exercises, and she has already tried the diet changes recommended in the lay press. She had a hysterectomy 20 years ago due to a complication of childbirth. There is no family history of breast cancer or thromboembolic disease. You agree with her that she is most likely experiencing marked symptoms of menopause. She asks about hormone replacement therapy because her mother got relief from it. You review the recent evidence against it and discuss other options with her, but she insists on trying hormones. You give her some literature, refer her for a mammogram, ask the lab to measure FSH, and schedule a follow-up appointment. The mammogram is negative, and the FSH confirms menopause. She still wants to try hormone therapy, so you start her on low-dose estrogen. She comes back in three months and reports feeling much better and thanks you for the estrogen. You remind her that hormone therapy can increase her risk of death, but she still wants the hormone therapy. It has been almost five years since you last checked her lipids, so you ask her to return in 3-4 months for fasting lipids. The HDL and non-HDL cholesterol are acceptable, but the triglycerides have markedly increased from 107 mg/dL to 820 mg/dL. Which of the following should be done at this time? A) Stop treating her symptoms despite her preferences. B) Hold the estrogen and resume once the triglycerides are normal again. C) Continue the estrogen and recheck lipids in 6-12 months. D) Switch from oral estrogen to a transdermal patch. E) Switch from estrogen to black cohosh from her local health food store

D. Switch from oral estrogen to a transdermal patch. Explanation This question requires you to know the effect of different modes of estrogen administration on triglyceride levels. Several recent trials involving hormone replacement therapy provide compelling evidence against its routine use in menopause. The patient in this scenario accepts the risk and wants hormone therapy to relieve her symptoms. Unfortunately, the estrogen induces hypertriglyceridemia. She is already on a low dose of estrogen, so lowering the dose any further is not likely to help—and may not provide relief from her symptoms. Her triglyceride response is likely to continue if the estrogen is continued or to recur if the estrogen is temporarily withheld. Alternative medicine is not likely to give her the same degree of relief from her symptoms as the estrogen, and she is unlikely to agree with no treatment at all. Switching her to transdermal estrogen should resolve her triglyceride issue. This is a notorious problem with oral estrogen because of the first-pass effect in the liver.


Ensembles d'études connexes

Regulations and Facilities-RATS Final Exam Review

View Set

Health Assessment Questions (practice)

View Set

Lecture 17: Aging: Physiological and evolutionary explanations (Learning Objectives and Questions)

View Set